2 Pathogenesis of Bacterial Diseases (78) Flashcards

1
Q

Case
A 57-year-old male farmer who resides in a rural area in Thailand presents with a 2-day history of a sudden onset of fever with chills, headache, lethargy, and confusion. His medical history is significant for diabetes mellitus, which was diagnosed 6 months ago; however, he is noncompliant to therapy. He has had recurrent subcutaneous abscesses for the past 3 months. The patient appears to be critically ill; he is disoriented, his temperature is 39°C, pulse is 140/minute, and blood pressure is 70/40 mmHg. The clinical diagnosis is septic shock; blood samples are sent for culture. Within 24 hours, the blood cultures grow slender, Gram-negative bacillus with bipolar staining and rounded ends, in addition to having a safety pin appearance. The bacterium is motile and produces small, smooth, non-hemolytic colonies on blood agar; the colonies become dry and wrinkled within 48 hours. The isolate grows on MacConkey’s medium and produces small, lactose-fermenting colonies. It is oxidase-positive, grows at 42°C, liquefies gelatin, and oxidizes various carbohydrates (e.g., glucose and lactose).

What is the most likely blood culture isolate?

1 Burkholderia pseudomallei
2 Yersinia pestis
3 Klebsiella pneumoniae
4 Pseudomonas aeruginosa
5 Burkholderia mallei

A

Burkholderia pseudomallei

Though all of the bacteria listed can produce septicemia, the characteristics of the isolate are suggestive of B. pseudomallei. The location and occupation of the patient and his clinical history are also suggestive of a B. pseudomallei infection. Burkholderia pseudomallei (Pseudomonas pseudomallei) is a soil saprophyte which causes melioidosis, a zoonotic disease endemic in many southeast Asian countries and in tropical Australia. It is a common cause of community-acquired sepsis in these areas. The bacterium is often found in tropical wet soil (e.g., in rice paddy fields). Farming and gardening necessitate prolonged or recurrent contact with soil; this can lead to infection, especially in individuals with risk factors (e.g., uncontrolled diabetes). Other risk factors include thalassemia, chronic renal failure, chronic lung disease, and excessive alcoholism. Human infection usually occurs through skin abrasions or by inhalation. Contamination of drinking water supplies has also been implicated in outbreaks of melioidosis in Australia. The disease may occur in the form of acute septicemia, subacute typhoid-like disease, or pulmonary infection (the most common form resembling tuberculosis). In the chronic form, there may be multiple suppurative foci with abscess formation in the skin, subcutaneous tissue, or internal organs. Skin and soft tissue infections may be sources of systemic infection; they can lead to an acute septicemic form of melioidosis. Acute melioidosis has a high fatality rate.
Long latency and reactivation can occur, as the bacillus can survive intracellularly in the reticuloendothelial system. Virulence factors of B. pseudomallei that allow evasion of killing mechanisms by the phagocytes have been characterized, and the genes associated with virulence have been identified. The virulence factors include various secretory antigens and cell-associated antigens (e.g., glycocalyx capsular polysaccharide, lipopolysaccharide, flagellin protein, pili, and a siderophore).
Laboratory diagnosis of melioidosis depends on the isolation of B. pseudomallei from various clinical specimens (e.g., blood, pus, and sputum) or by serology. Ashdown’s medium is one of the selective media used for the isolation of the bacterium from clinical specimens that are obtained from nonsterile body sites; it contains gentamicin as a selective agent. Serological tests for the diagnosis of melioidosis include an enzyme-linked immunosorbent assay (ELISA) for IgM and IgG antibodies and an indirect hemagglutination test. Rapid tests such as immunochromogenic card tests for IgM and IgG and DOT immunoassay have been developed, but not extensively tested. A monoclonal antibody latex agglutination test and Immunofluorescence test are reported to be useful for the detection of the B. pseudomallei antigen. A polymerase chain reaction (PCR)-based test has also been developed for the identification of B. pseudomallei.
Ceftazidime and imipenem are the drugs of choice. Combination therapy, along with cotrimoxazole, tetracycline, amoxicillin clavulanate, or chloramphenicol, has also been found to be beneficial. Prolonged treatment may be required. Melioidosis is a rare disease in the United States; however, cases of imported melioidosis and infection in exposed laboratory workers have been reported.
A safety pin appearance is a morphological feature of Yersinia pestis as well. This bacterium can be differentiated from B. pseudomallei; it is nonmotile, oxidase-negative, and produces non-lactose-fermenting, colorless colonies on MacConkey’s medium. Yersinia pestis produces plague, a zoonotic disease, which is transmitted by rat fleas. Septicemic plague is one of the clinical forms, either as a terminal event in bubonic or pneumonic plague, or as a primary form.
Klebsiella pneumoniae grows on MacConkey’s medium and produces large, lactose-fermenting mucoid colonies. The bacterium is nonmotile and oxidase-negative. It causes pneumonia, urinary infection, other pyogenic infections, and septicemia. Klebsiella pneumoniae and Yersinia pestis are both nonmotile and oxidase-negative.
Pseudomonas aeruginosa is a ubiquitous bacterium. It is one of the most common agents of nosocomial infections, including septicemia. It possesses characteristics that are similar to B. pseudomallei.; it is an oxidase-positive, actively motile, slender Gram-negative bacterium. It does not show the safety pin morphology. It produces colored colonies due to the production of pigments (e.g., pyocyanin, fluorescin, or pyorubrin). It is often hemolytic on blood agar and on MacConkey’s medium and produces non-lactose-fermenting colonies. It utilizes glucose (not lactose) oxidatively.
Burkholderia mallei (Pseudomonas mallei) resembles B. pseudomallei, but differences are in nonmotility, the inability to form acid from lactose, and the inability to liquefy gelatin. It is the causative agent of glanders, a disease primarily of equine animals. Human infection acquired from infected animals is rare and occupational; it is generally found in hostlers and veterinarians. Laboratory cultures are highly infectious.
B. pseudomallei, B. mallei, and Yersinia pestis are considered to be potential biological weapons.

How well did you know this?
1
Not at all
2
3
4
5
Perfectly
2
Q

Case
A 30-year-old male patient presents with a 1-hour history of severe nausea and vomiting. Before falling ill, he was at a party where he ate pudding and other food. Physical examination reveals an unremarkable temperature with mild diffuse tenderness of the abdomen. The organism isolated is a Gram-positive coccus that occurs in grape-like clusters; it is catalase- and coagulase-positive, and it forms a golden-yellow colony on agar.

What toxin released by the causative organism is responsible for the patient’s symptoms?

1 Alpha toxin
2 Exfoliatin toxin
3 Enterotoxin
4 Leukocidin
5 Toxic shock syndrome toxin (TSST-1)

A

Enterotoxin

Enterotoxin produced by Staphylococcus aureus is an exotoxin; it is responsible for this patient’s condition. The clinical presentation and the laboratory findings (short incubation period of 1-6 hours with predominant emesis) suggest Staphylococcal food poisoning. S. aureus organisms are Gram-positive cocci that occur in grape-like clusters. They are catalase- and coagulase-positive, and they form golden-yellow colonies on agar. Staphylococcal food poisoning results from the ingestion of preformed enterotoxins on food contaminated with S. aureus. Bacteria growing in carbohydrates and meat products produce enterotoxins that, upon ingestion, diffuse into the circulation and cause emesis by stimulating the vomiting center in the central nervous system.
Toxic shock syndrome toxin (TSST-1) has superantigen activity, and it causes life-threatening toxic shock syndrome when expressed systemically. The clinical presentation includes fever, hypotension, and diffuse macular erythema; at least three organ systems (gastrointestinal, renal, hepatic, musculoskeletal, nervous) are involved.
Exfoliatin toxin (ET) elaborated by S. aureus causes scalded skin syndrome, manifested as widespread blistering and loss of the epidermis, revealing a red base. ETA and ETB are the two antigenically distinct forms of the toxin. ET has esterase and protease activity, which targets a protein involved in maintaining epidermal integrity, causing separation of the epidermis.
Leukocidin is a membrane-damaging toxin expressed by S. aureus that acts on polymorphonuclear leukocytes.
Alpha toxin (alpha-hemolysin), the most potent membrane-damaging toxin of S. aureus, causes hemolysis.

How well did you know this?
1
Not at all
2
3
4
5
Perfectly
3
Q

Case
In the medical intensive care unit (MICU) of a 300-bed hospital, 5 patients developed ventilator-associated pneumonia (VAP) over a period of 1 month. All patients who developed VAP had been in the ICU for more than 7 days. In the ICU, third generation cephalosporins were commonly used for empiric Gram-negative coverage and vancomycin for Gram-positive coverage.

Purulent respiratory secretions collected from the patients were processed in the microbiology laboratory. Smear examination showed numerous polymorphs and a high predominance of intra- and extracellular Gram-negative diplococcal and coccobacillary forms. Cultures of all samples yielded pure growth of Gram-negative bacteria with identical characteristics. All the 5 isolates belonged to the same bacterial species, indicating the possibility of an outbreak. The characteristics that helped presumptive identification were microscopic appearance as Gram-negative diplococcal and coccobacillary forms (1-1.5microns x 1.5-2.5 microns in size), absence of motility, ability to grow on ordinary culture media, production of non-hemolytic large, whitish, smooth, round colonies on sheep blood agar; negative oxidase, positive catalase; absence of fermentative activity; production of acid from glucose oxidatively; and inability to reduce nitrate. All isolates had identical antibiotic susceptibility patterns. Multidrug-resistance was observed, the isolates being resistant to most of the beta-lactam antibiotics including third generation cephalosporins and most of the aminoglycosides, including gentamicin and tobramicin. Susceptibility was seen to imipenem and amikacin.

What bacteria is the most likely agent of the VAP outbreak in the MICU?

1 Serratia marcescens
2 Klebsiella pneumoniae
3 Pseudomonas aeruginosa
4 Acinetobacter baumannii
5 Moraxella catarrhalis

A

Acinetobacter baumannii

The characterstics of the isolates are in favor of Acinetobacter baumannii. Acinetobacter baumannii is an opportunistic pathogen. Nosocomial outbreaks including outbreaks of ventilator-associated pneumonia (VAP) caused by this bacterium are increasingly reported. Outbreaks are usually associated with multidrug-resistant strains. A. baumannii occurs as a saprophyte in soil and water and is often present in the hospital environment. It can occasionally be isolated from human skin and mucous membranes. The bacterium can survive on dry inanimate surfaces for months.
VAP is defined as nosocomial pneumonia in a patient on mechanical ventilatory support (by endotracheal tube or tracheostomy) for a period of 48 hours or more. In mechanically ventilated patients, aspiration of organisms colonizing the orophrynx is the main route of entry of bacteria to the lower respiratory tract. The lower respiratory tract can become colonized by potential pathogens from endogenous and exogenous sources. Colonization and infection by endogenous organisms like methicillin sensitive staphylococcus, S. pneumoniae, and unencapsulated Haemophilus influenzae are seen within the initial 4-5 days. Early-onset VAP is caused mainly by these bacteria. Prior antibiotic therapy and prolonged intubation favors colonization and infection by multidrug-resistant Gram-negative bacteria. A patient who has been under mechanical ventilation for more than 7 days and receiving broad spectrum antibiotics is at the risk of getting late-onset VAP caused by more resistant exogenous flora like Pseudomonas aeruginosa, Acinetobacter baumannii, and methicillin-resistant Staphylococcus aureus (MRSA). Outbreaks can originate from common environmental sources like the room humidifiers and vaporizers or any other device. Transmission may occur by the hands of health care workers if the hand washing policy is not strictly followed. During an outbreak investigation, different possible sources are screened for the bacterium and all isolated strains are identified by molecular typing.
Antibiotic resistance in acinetobacter is acquired by exposure to antibiotics rather than being inherent. In the hospital environment, chromosomal cephalosporinase production facilitated by selective pressure for beta-lactam use confers cephalosporin resistance to Acinetobacters. Carbapenems (imipenem and meropenem) are widely used for the treatment of infection by such strains. Carbapenem resistance may develop as a result of altered outer membrane proteins impairing permeability of cell membrane or alterations in the penicillin-binding proteins. Recently, outbreaks caused by carbapenem-resistant clones of Acinetobacter baumannii have been reported from various countries including the U.S. Carbapenem resistance due to production of specific carbapenem-hydrolyzing beta lactamases (carbapenemases) has been detected as a characteristic of these clones. Colistin is found to be of use in treating infections by the carbapenem-resistant strains. Ampicillin-sulbactam is considered as a cost-effective therapeutic option for treatment of severe infections due to multidrug-resistant A. baumannii. In vitro susceptibility of multidrug-resistant A.baumannii to a new glycylcycline antibiotic, Tigecycline, has been recently reported.
Serratia marcescens is a Gram-negative bacillus widely distributed in nature and belongs to enterobacteriaceae. It has emerged as an important nosocomial pathogen in the last few years. It can be differentiated from Acinetobacter by its ability to reduce nitrate to nitrite and fermentation of carbohydrates. Some strains produce a pigment prodigiosin that gives a red color to the colonies. Presence of pigment-producing strains in sputum can give a red color to the sputum resembling hemoptysis. Nosocomial colonization and outbreaks of infections are mostly caused by multidrug-resistant strains.
Klebsiella pneumoniae is a short Gram-negative, capsulated, nonmotile rod capable of causing a wide variety of community as well as hospital-acquired infections. Multidrug-resistant strains occur in the hospital environment and cause nosocomial infections including VAP. It belongs to enterobacteriaceae and, unlike A. baumannii, reduces nitrate to nitrite and ferments various carbohydrates with production of acid and gas.
Pseudomonas aeruginosa, a ubiquitous Gram-negative bacillus, is one of the most common nosocomial pathogens with intrinsic as well as acquired resistance to various antibiotics. The bacterium can be differentiated from Acinetobacter by its active motility, positive oxidase test, and production of pigments.
Moraxella catarrhalis is a member of the normal flora of the upper respiratory tract. It is usually associated with opportunistic community-acquired infections like bronchitis, pneumonia, sinusitis, and otitis media. Morphologically it resembles Acinetobacter baumannii, is a non-fermenter, and it can be differentiated by positive oxidase test.

How well did you know this?
1
Not at all
2
3
4
5
Perfectly
4
Q

A 2-month-old baby was brought to the pediatric clinic situated in one of the developing countries in South America. He was diagnosed with congenital heart disease. The infant’s mother had history of mild febrile illness with rashes during the first trimester of her pregnancy, suggestive of rubella infection. Which of the following tests is most likely to be used for routine diagnosis of congenital rubella infection in the baby?

1 Isolation of rubella virus from urine using vero cells
2 Isolation of rubella virus from throat secretions using rabbit kidney cells
3 Detection of rubella RNA in serum by PCR-based test
4 Demonstration of rubella-specific IgG in serum
5 Demonstration of rubella-specific IgM in serum

A

Demonstration of rubella-specific IgM in serum

Congenital rubella infection is routinely diagnosed by serological demonstration of rubella-specific IgM antibodies. The presence of IgM antibodies indicates immune response in the fetus, whereas presence of IgG antibodies in a 2-month-old infant may be due to passive transfer of maternal antibodies. Virus isolation is not routinely used for diagnosis because of the difficulties and delay involved.
PCR-based test for detection of rubella virus RNA is not available for routine diagnostic use.
Rubella or German measles is a mild exanthematous fever characterized by transient macular rash and lymphadenopathy. The infection is harmful only when it occurs in a pregnant woman especially in the first trimester of pregnancy. The extent of teratogenic effects produced by the virus depends on the timing of infection. The earlier the maternal infection, the more fetal damage, resulting in death or congenital malformations of the fetus. The classical triad of congenital rubella syndrome consists of cataract, deafness, and cardiac defects. Intellectual disability, hepatosplenomegaly, thrombocytopenic purpura, and central nervous system involvement are other manifestations constituting expanded rubella syndrome.
Rubella virus belongs to Togaviridae family and is the only member of genus Rubivirus. The viral particle is 50-70 nm in diameter with single stranded RNA genome surrounded by an envelope carrying hemagglutinin peplomers.
Laboratory diagnostic methods for rubella include virus isolation and serology. Serological diagnosis is routinely used. Enzyme Liked Immunosorbent Assay (ELISA) is used for detection of IgM and IgG antibodies. In a pregnant woman with suspected rubella, IgM antibody alone without IgG means current acute infection. IgG antibody alone without IgM is suggestive of past infection or vaccination and indicates immunity.
In congenital rubella, viral excretion may last for several months after birth. The virus may be isolated from a variety of sources such as urine, throat secretions, bone marrow, or cerebrospinal fluid. Rabbit kidney or Vero cells are used for virus isolation.
The virus grows better if cultures are incubated at lower temperature, around 33-35°C. Cytopathogenic effect produced by rubella virus is inconspicuous. Immunofluorescence may be used for detection of viral antigens in cell cultures 3-4 days after inoculation.
Detection of IgG antibodies can be used for diagnosing congenital rubella infection in infants more than 6 months old.
Reverse Transcriptase Polymerase chain reaction (RT-PCR) assay has been used for detection of rubella RNA in serum samples during epidemiological surveillance for congenital rubella syndrome. RT-PCR assay has also been used to detect rubella genome in oral fluids, throat swabs and tissue samples. Acute cases within 2 days after onset of symptoms could be diagnosed by this assay, even before IgM response reached detectable levels. The test has been successfully used for diagnosis of prenatal congenital rubella infection by detection of rubella genome in amniotic fluid.
Since 1969, the use of attenuated live rubella vaccine has considerably reduced the incidence of congenital rubella in developed countries and some developing countries.

How well did you know this?
1
Not at all
2
3
4
5
Perfectly
5
Q

Case
A 27-year-old male accident victim with a head injury is admitted to the ICU and kept on mechanical ventilatory support. On the seventh day after admission, he is clinically diagnosed with pneumonia. Blood samples and lower respiratory secretions are submitted to the laboratory for culture; empiric antimicrobial therapy is started.

What is the most likely etiologic agent of pneumonia in this patient?

1 Streptococcus pneumoniae
2 Klebsiella pneumoniae
3 Mycoplasma pneumoniae
4 Moraxella catarrhalis
5 Haemophilus influenzae

A

Klebsiella pneumoniae

The correct response is Klebsiella pneumoniae.
The patient had been on mechanical ventilation for >5 days when he developed pneumonia; by definition, he has nosocomial late-onset ventilator-associated pneumonia (VAP). The main etiological agents of late-onset VAP consist of antibiotic-resistant gram-negative bacilli and methicillin-resistant Staphylococcus aureus. The most common gram-negative bacilli associated with late-onset VAP are Pseudomonas, Klebsiella, Enterobacter, Serratia, and Acinetobacter species. Often, more than one organism is involved.
From the given list of bacteria, the most likely etiological agent of pneumonia in this patient is Klebsiella pneumoniae. Multi-drug resistant strains of Klebsiella pneumoniae occur in the hospital environment and are known to be important agents of nosocomial infections, including late-onset VAP.
Streptococcus pneumoniae, Haemophilus influenzae, and Moraxella catarrhalis are generally associated with community-acquired pneumonia, but colonization and infection by these endogenous bacteria can occur within the initial 4-5 days of ventilatory support (resulting in early-onset VAP).
Mycoplasma pneumoniae is an important agent of community-acquired pneumonia. Recently, the possible involvement of this organism in early-onset VAP has been suggested.

How well did you know this?
1
Not at all
2
3
4
5
Perfectly
6
Q

Staphylococcus epidermidis is normally found on the skin but may also be found in surface wounds and abrasions without being the true etiologic agent of the infection. In this case, the organism is said to:

1 Infect the wound.
2 Parasitize the wound.
3 Colonize the wound.
4 Infiltrate the wound.
5 Contaminate the wound.

A

Colonize the wound.

The correct answer is colonize the wound. Both coagulase-positive and coagulase-negative staphylococci are normal flora on the skin of healthy individuals, and their presence in a surface lesion must be carefully considered in light of other clinical factors. Generally, the coagulase-negative staphylococci, including S. epidermidis, are considered only harmless colonizers, especially in the presence of other normal skin flora. There are cases, however, where the coagulase-negative staphylococci are considered etiologic agents, especially in surgical wound infections where they may be the only agent cultured.
The term “contamination of a wound” may or may not imply true infection and is not often used to describe the role of a particular organism in a lesion. Infection implies multiplication and spreading of the agent, usually with accompanying exudate of polymorphonuclear leukocytes. Infiltration and parasitization are not usually used to describe the relationship of an organism to a particular wound site, although organisms may in fact infiltrate into tissue thereby causing infection.

How well did you know this?
1
Not at all
2
3
4
5
Perfectly
7
Q

Diphtheria toxin, produced by Corynebacterium diptheria acts by

1 Inhibiting protein synthesis by ADP-ribosylation of elongation factor 2 (EF-2)
2 Inducing TNF
3 Inducing interleukin-1 production
4 Degrading cell walls

A

Inhibiting protein synthesis by ADP-ribosylation of elongation factor 2 (EF-2)

Diphtheria toxin, produced by Corynebacterium diptheria , inhibits protein synthesis by ADP-ribosylation of elongation factor 2 (EF-2). This activity depends on two functions mediated by different domains of the molecule. Fragment A, a 22,000 molecular weight peptide at the amino-terminal end is the enzyme that catalyzes the transfer of ADP-ribose from nicotinamide adenine dinucleotide (NAD) to EF-2, thereby inactivating it. The ADP-ribosylation freezes the translocation complex, and protein synthesis stops.

How well did you know this?
1
Not at all
2
3
4
5
Perfectly
8
Q

Tetanus toxin acts by

1 Its neurotoxic effect in preventing the release of the inhibitory neurotransmitter glycine
2 Blocking the release of acetylcholine
3 Stimulating adenylate cyclase activity
4 Increasing cAMP production

A

Its neurotoxic effect in preventing the release of the inhibitory neurotransmitter glycine

Tetanus toxin, produced by Clostridium tetani , is a neurotoxin that prevents release of the inhibitory neurotransmitter glycine. This causes muscle spasms. The toxin (tetanospasmin) is composed of two polypeptide subunits encoded by plasmid DNA. The heavy chain of the polypeptide binds to gangliosides in the membrane of the neuron; the light chain exerts the toxic activity.

How well did you know this?
1
Not at all
2
3
4
5
Perfectly
9
Q

Verotoxin

1 Is an endotoxin produced by E.coli (O157:H7 serotype)
2 Is an exotoxin produced by E.coli (O157:H7 serotype)
3 Is produced by Streptococcus viridans
4 Is produced by Clostridium tetani

A

Is an exotoxin produced by E.coli (O157:H7 serotype)

Verotoxin is an exotoxin produced by strains of E.coli with the O157:H7 serotype. These strains cause bloody diarrhea and are the cause of outbreaks associated with eating undercooked hamburgers. The toxin is named for its toxic effect on Vero (monkey) cells in culture.

How well did you know this?
1
Not at all
2
3
4
5
Perfectly
10
Q

The heat-labile enterotoxin produced by E.coli causes diarrhea by

1 Stimulating adenylate cyclase activity in cells in the small intestine
2 Stimulating guanylate cyclase
3 Blocking protein synthesis
4 Blocking release of acetylcholine

A

Stimulating adenylate cyclase activity in cells in the small intestine

The heat-labile enterotoxin produced by E.coli causes diarrhea by stimulating adenylate cyclase activity in cells in the small intestine. The resulting increase in the concentration of cyclic adenosine monophosphate (cAMP) causes excretion of the chloride ion, inhibition of sodium ion absorption, and significant fluid and electrolyte loss into the lumen of the gut.

How well did you know this?
1
Not at all
2
3
4
5
Perfectly
11
Q

The botulinum toxin acts by:

1 Blocking the release of acetylcholine at the synapse and thereby producing paralysis
2 Affecting cGMP activity
3 Blocking protein synthesis
4 Activating the complement cascade

A

Blocking the release of acetylcholine at the synapse and thereby producing paralysis

Botulinum toxin produced by Clostridium botulinum is a neurotoxin that blocks the release of acetylcholine at the synapset, producing paralysis. The genes for this toxin are encoded by a temperate bacteriophage. It is composed of 2 polypeptide subunits held together by disulfide bonds and one of the subunits binds to a receptor on the neuron.

How well did you know this?
1
Not at all
2
3
4
5
Perfectly
12
Q

Toxic shock syndrome toxin (TSST) is produced by

1 Bordetella pertussis
2 Staphylococcus aureus
3 Bacillus anthracis
4 Escherichia coli

A

Staphylococcus aureus

Toxic shock syndrome toxin (TSST) is produced by certain strains of Staphylococcus aureus. TSST binds directly to class II major histocompatibility (MHC) proteins without intracellular processing. This complex interacts with the β-chain of the T cell receptor of many helper T cells and causes the release of large amounts of interleukins, which then produce many of the signs of the toxic shock.

How well did you know this?
1
Not at all
2
3
4
5
Perfectly
13
Q

The ability of an organism to produce pathologic changes or disease in the host is known as

1 Virulence
2 Inflammation
3 Toxigenicity
4 Pathogenicity
5 Invasiveness

A

Pathogenicity

The capacity of pathogens to cause disease depends on their abilities to infect the host or protect themselves against the body’s defenses, to invade and multiply in tissues and to cause damage to the tissue.
Virulence is the measure of degree of pathogenicity and can be defined as the sum of infectivity, invasiveness and toxigenicity.
Inflammation is a nonspecific pathologic process consisting of a dynamic complex of cytologic and histologic reactions that occur in response to an injury or abnormal stimulation by a physical, chemical, or biologic agent.
Toxigenicity is the ability of the organism to produce toxins.
Invasiveness is the ability of the organism to penetrate and grow in the host tissue.

How well did you know this?
1
Not at all
2
3
4
5
Perfectly
14
Q

Substances that stick to a microorganism surface and make it more attractive to phagocytic cells are known as:

1 Interferons
2 Bacteriocins
3 Lysozymes
4 Opsonins
5 Phagosomes

A

Opsonins

Opsonins stick to the surface of a microorganism and enhance phagocytosis.
Interferons are the class of small, antiviral glycoproteins produced by cells infected with an animal virus.
Bacteriocins are proteins produced by certain bacteria that can kill other bacteria.
Lysozyme is a digestive enzyme found in lysozymes, tears, and other body fluids and is destructive to bacterial cell walls.
Phagosome is a membrane bound vesicle containing an ingested particle and is found in phagocytic cells.

How well did you know this?
1
Not at all
2
3
4
5
Perfectly
15
Q

Select the most true statement relating to exotoxins

1 Produced by gram negative bacteria only
2 Lipopolysaccharide in composition
3 Antibodies do not neutralize toxicity
4 Never contain ketodeoxyoctonate (KDO)
5 Toxicity resides in lipid A component

A

Never contain ketodeoxyoctonate (KDO)

Exotoxins are protein produced by gram negative and gram positive bacteria. Antibodies neutralize their toxicity and never contain ketodeoxyoctonate (KDO). Toxicity is due to biologic action of protein. Toxicity resides in the lipid A component in endotoxins.

How well did you know this?
1
Not at all
2
3
4
5
Perfectly
16
Q

Endotoxins are

1 Mostly heat labile
2 Always highly toxic in microgram quantities
3 Known as lipopolysaccharides
4 Used to form toxoids
5 Produced by gram positive bacteria only

A

Known as lipopolysaccharides

Endotoxins are lipopolysaccharide portion of the cell wall produced by gram negative bacteria only. They are heat stable and cannot be used to form toxoids. Endotoxins are toxic in milligram quantities, while most exotoxins are toxic in microgram quantities.
Endotoxin stimulates the fever center in the hypothalamus and causes hypotension. It initiates coagulation that can result in intravascular coagulation.

How well did you know this?
1
Not at all
2
3
4
5
Perfectly
17
Q

The enzyme that enables an organism to clot plasma and form a fibrin coat is

1 Streptokinase
2 Hyaluronidase
3 Staphylokinase
4 Coagulase
5 Collagenase

A

Coagulase

Coagulase is the exoenzyme produced by Staphylococcus aureus, which causes citrated blood plasma to clot in the laboratory. However, kinases dissolve fibrin clots, thus enabling the organism to invade and spread throughout the body.
Hyaluronidase enables pathogen to spread through connective tissue by breaking down hyaluronic acid, the “cement” that holds tissue cells together.
Collagenase breaks down collagen, the supportive protein found in tendons.

How well did you know this?
1
Not at all
2
3
4
5
Perfectly
18
Q

The ability of pathogens to invade, infect, cause damage and produce disease in the host is termed as

1 Pathogenicity
2 Toxigenicity
3 Virulence
4 Infection
5 Attenuation

A

Virulence

Virulence is the ability of a pathogen to invade, infect, cause damage and produce disease in the host.
Pathogenicity is the ability of an organism to produce pathologic changes or disease in the host.
Toxigenicity is the ability of the organism to produce toxins.
Infection occurs when a pathogenic microbe is able to multiply in the tissues where it lodges.
Attenuation is the process by which the microorganism is rendered less pathogenic or avirulent.

How well did you know this?
1
Not at all
2
3
4
5
Perfectly
19
Q

Diphtheria toxin, that is produced by the bacterium Corynebacterium diphtheriae, exerts its action by which of the following?

1 Blocking release of acetylcholine
2 Blocking release of inhibitory neurotransmitters
3 Inhibiting protein synthesis by ADP ribosylation of elongation factor 2
4 Acting as an adenylate cyclase
5 An unknown mechanism

A

Inhibiting protein synthesis by ADP ribosylation of elongation factor 2

Diphtheria toxin binds to the receptors on the cell membranes via a region near its carboxyl end. This is followed by transport across the cell membrane, a proteolytic nick and reduction of the disulfide bond. This in turn releases active fragment A, which inactivates EF-2. The following chemical reaction occurs:
EF-2 + NAD —–> EF-2-ADP-ribose + Nicotinamide + H+
Clostridium botulinum exotoxin blocks release of acetylcholine. Clostridium tetani exotoxin interferes with the release of inhibitory neurotransmitters such as glycine and gamma-aminobutyric acid. One of the toxins of Bacillus anthracis is an adenylate cyclase

How well did you know this?
1
Not at all
2
3
4
5
Perfectly
20
Q

One of the biological effects of endotoxins is fever. This is caused due to release of

1 Bradykinin
2 Interleukin-1
3 Alternative pathway of complement C3a and C5a
4 Hageman factor
5 Virus

A

Interleukin-1

Interleukin-1 is released by macrophages; it acts on the hypothalamic temperature regulatory center and causes fever. Bradykinin causes hypotension (shock). The alternative pathway of complement C3a and C5a causes inflammation. Hageman factor initiates coagulation. Endotoxins do not release viruses.

How well did you know this?
1
Not at all
2
3
4
5
Perfectly
21
Q

Aflatoxin is a toxin produced by an organism that is not generally invasive in humans although the toxin itself may be extremely dangerous. What is the organism most likely to produce this toxin?

1 Acinetobacter baumanii
2 Agrobacterium radiobacter
3 Aspergillus flavus
4 Actinomyces naeslundii
5 Acremonium sp

A

Aspergillus flavus

Aflatoxin is a fungal toxin that is produced by Aspergillus flavus and Claviseps purpurae. The other organisms listed, with the exception of Acremonium sp., are bacteria and are not producers of exotoxins. Normally these aflatoxins make their way into humans through their contamination of various foods. Aflatoxin is actually named from its major source, Aspergillus flavus.

How well did you know this?
1
Not at all
2
3
4
5
Perfectly
22
Q

Ergot poisoning is an infrequent but significant contributor to illness due to the toxin ergotamine. The most likely source of this toxin would be

1 Bacillus cereus toxin contaminating cooked rice
2 Staphylococcus aureus toxin contaminating cream-based foods
3 Histoplasma capsulatum toxin contaminating herbs
4 Entamoeba histolytica toxin contaminating water
5 Claviceps purpurae toxin contaminating wheat and rye products

A

Claviceps purpurae toxin contaminating wheat and rye products

Ergotamine is a fungal toxin produced by Claviceps sp. Humans are poisoned by eating contaminated grain such as wheat and rye.

How well did you know this?
1
Not at all
2
3
4
5
Perfectly
23
Q

Of the organisms listed, which one produces a neurotoxin?

1 Bacillus cereus
2 Clostridium perfringens
3 Escherichia coli
4 Clostridium botulinum
5 Vibrio cholerae

A

Clostridium botulinum

C. botulinum produces a potent neurotoxin that causes botulism, a potentially life-threatening food-borne illness exhibiting neurologic symptoms. The other organisms listed produce an enterotoxin that results in intestinal symptoms also resulting from ingestion.

How well did you know this?
1
Not at all
2
3
4
5
Perfectly
24
Q

The exotoxin responsible for this disease is produced by a noninvasive organism. The toxin activates the catalytic subunit of adenylate cyclase, resulting in persistent production of cAMP because the control reaction (GTP hydrolysis by GTPase) is no longer operative. The continuous synthesis of cAMP provokes the movement of water and electrolytes across the intact epithelial cells into the lumen of the small intestine producing diarrhea. The most likely diagnosis resulting from this mechanism of action is:

1 Food poisoning
2 Cholera
3 Bacillary dysentery
4 Amebic dysentery
5 Pseudomembranous colitis

A

Cholera

The toxin described is unique to Vibrio cholerae . Other enterotoxins act with a different mechanism

How well did you know this?
1
Not at all
2
3
4
5
Perfectly
25
Q

Endotoxin is the

1 Protein component of the cell well of gram negative bacteria
2 Protein component of the cell wall of gram positive bacteria
3 Lipopolysaccharide component of the outer membrane of gram positive bacteria
4 Lipopolysaccharide component of the outer membrane of gram negative bacteria
5 Lipoprotein component of the membrane of both gram positive and gram negative bacteria

A

Lipopolysaccharide component of the outer membrane of gram negative bacteria

Only gram negative bacteria have an outer membrane, and it is this component that is referred to as endotoxin. The toxicity of endotoxin varies greatly among these organisms and resides in the Lipid A portion.

How well did you know this?
1
Not at all
2
3
4
5
Perfectly
26
Q

The limulus lysate test is used in both industry and medicine as a sensitive procedure to detect the presence of even picogram amounts of

1 Viral DNA
2 Bacterial endotoxin
3 Bacterial exotoxin
4 Parasitic exoantigens
5 Fungal exotoxins

A

Bacterial endotoxin

A commonly used screening test for endotoxin is the limulus lysate test. This test is based on the observation that amebocytes from the horseshoe crab, limulus polyphemus, clot in the presence of even picogram amounts of bacterial endotoxin. The limulus test has formerly been used in the clinical laboratory to screen spinal fluid for endotoxin as an indication of the presence of gram negative bacteria. It is not a routine procedure but may be available at reference laboratories.

How well did you know this?
1
Not at all
2
3
4
5
Perfectly
27
Q

Which of the following organisms would be considered invasive

1 E. coli O157:H7
2 Vibrio cholerae
3 Corynebacterium diphtheriae
4 Salmonella typhi
5 Shigella dysenteriae

A

Salmonella typhi

All the organisms listed, with the exception of S. typhi, produce disease via a toxin. S. typhi invade through the intestinal mucosa, beyond the lamina propria, and ultimately into the blood stream and back into the gall bladder where this organism may remain for the life of the patient unless the gall bladder is removed.

How well did you know this?
1
Not at all
2
3
4
5
Perfectly
28
Q

To quickly determine if an invasive diarrheal pathogen or a toxin producer is present in a stool specimen, one may

1 Stain the specimen and look for white blood cells, which would indicate an invasive process
2 Culture the specimen and observe for beta-hemolytic isolates, which would indicate toxin producers
3 Perform a limulus lysate test on the specimen, which would detect endotoxin
4 Treat the specimen with alcohol to kill all but spore-forming bacteria, then culture for the presence of these agents
5 Stain a pure culture isolate with the flagella stain, which would indicate invasiveness in the presence of these flagella

A

Stain the specimen and look for white blood cells, which would indicate an invasive process

The presence of white blood cells in a stool specimen often indicates an invasive process in cases of diarrhea; whereas, toxin-mediated diarrhea is usually not accompanied by white blood cells in the feces. Beta-hemolysis is not an indicator of toxicity, nor do flagella make an organism invasive. Because the feces is heavily colonized by gram negative bacteria, one would expect the limulus lysate test (if it were performed) to be positive; however, this test is never performed on stool specimens because all tests would be positive. Alcohol treatment of stool can be used to assist in the culture of Clostridioides difficile by killing most vegetative cells and leaving bacterial spores unharmed. However, this procedure still does not indicate a mechanism of pathogenicity.

How well did you know this?
1
Not at all
2
3
4
5
Perfectly
29
Q

In the laboratory, a sputum culture from an adult patient experiencing respiratory symptoms, revealed the presence of Streptococcus pneumoniae, Haemophilus influenzae, and Staphylococcus aureus, all in relatively low numbers. Because the patient may have pneumonia one of the organisms cultured, may likely be the etiologic agent. Other than initiating empiric therapy, what would you do next to help confirm your suspected diagnosis?

1 Ask the lab to confirm the identification of the organisms by either enzyme immunoassay tests or conventional biochemical tests
2 Order acute and convalescent sera, drawn for subsequent serologic testing, looking for a 4-fold rise in titer to one of the agents
3 Review the gram stain results to determine if the specimen was adequate, determine which morphotype predominated on the smear, then order a blood culture
4 Order a bronchoalveolar lavage specimen to confirm the results of sputum culture
5 Ask the laboratory to do a susceptibility test on all three isolates

A

Review the gram stain results to determine if the specimen was adequate, determine which morphotype predominated on the smear, then order a blood culture

Sputum culture remains a common request for patients with pneumonia, even though it is fraught with error. The specimen of choice for the diagnosis of pneumonia may be a blood culture, especially if the results of blood culture can correlate to the findings from sputum. The one tool available to the laboratory to validate the adequacy of the sputum specimen is the gram stain. The presence of squamous epithelial cells always indicates contamination with oropharyngeal flora. Compounding the problem of interpretation regarding the results of sputum culture, is the fact that all three of the organisms listed, S. pneumoniae, H. influenzae, and S. aureus are normal flora in the human respiratory, and they can also be pathogenic in the respiratory tract. Validation by gram stain and blood culture facilitates the interpretation of results.
Confirmation of identification or susceptibility test results would not help correlate culture results to patient outcome because the non-pathogen could be the one confirmed. Acute and convalescent sera are not used for testing these bacteria. A BAL specimen could be ordered if the sputum specimen had been inadequate or inappropriately collected, but such a procedure would add another 24-48 hours to the time for results.

How well did you know this?
1
Not at all
2
3
4
5
Perfectly
30
Q

For collecting a rectal specimen for the culture of Neisseria gonorrhoeae, the appropriate specimen will most likely be

1 A moistened calcium-alginate swab rubbed firmly over the entire perirectal area
2 A cotton-tipped swab inserted 1 inch into the anal canal and moved from side to side for about 15 seconds
3 A swab inserted into the rectum just beyond the anal sphincter in order to insure that an adequate amount of feces appears on the swab upon removal
4 A cotton-tipped swab used to firmly sample the external anal opening
5 A rectal swab is rarely recommended for the isolation of N. gonorrhoeae

A

A cotton-tipped swab inserted 1 inch into the anal canal and moved from side to side for about 15 seconds

N. gonorrhoeae is often isolated from rectal sources of homosexual men and from symptomatic women. The anal crypts must be firmly sampled by inserting the swab about an inch into the anal canal, and moving it from side to side. An external sample of the rectal or perirectal area is inadequate. Care should be taken to prevent too much feces from contaminating the swab tip during sampling.

How well did you know this?
1
Not at all
2
3
4
5
Perfectly
31
Q

The direct gram stain can be used effectively for the diagnosis of gonococcal urethritis in

1 Symptomatic men
2 Asymptomatic men
3 Both symptomatic and asymptomatic men
4 Asymptomatic women
5 Either symptomatic or asymptomatic women

A

Symptomatic men

When used properly by trained personnel, the direct gram stain is about 95% sensitive and 100% specific, for the diagnosis of gonococcal urethritis in symptomatic men. Culture of the urethra of these patients may be necessary only if the direct smear is equivocal or negative. The direct smear is not sensitive enough to detect gonococcal infection in asymptomatic men. Gram stained smears of exudate from women may be misleading since there are relatively large numbers of non-gonococcal Gram-negative rods residing as normal flora of the vagina.

How well did you know this?
1
Not at all
2
3
4
5
Perfectly
32
Q

The most dependable method for diagnosing gonorrhea in symptomatic women is
Answer Choices

1 The direct gram stain of an endocervical swab specimen
2 Culture of the cervix only
3 Culture of the cervix and rectum
4 Serum for IgG and IgM titers
5 Blood culture taken during a fever spike

A

Culture of the cervix and rectum

In symptomatic women, the direct gram stain has been shown to be only 60% sensitive. Cervical and rectal cultures should be performed to diagnose infection in these patients, as well as in asymptomatic women known to be contacts of infected men. In some cases, especially after therapy, rectal cultures may grow N. gonorrhoeae when the endocervical cultures are negative. Serology and blood specimens are not useful in cases other than disseminated gonococcal infection.

33
Q

The “Jembec” plate is designed specifically for the culture and transport of

1 Streptococcus pyogenes
2 Corynebacterium diphtheriae
3 Legionella pneumophila
4 Neisseria gonorrhoeae

A

Neisseria gonorrhoeae

The Jembec plate (John E. Martin Biological Environmental Chamber) is designed specifically for the culture and transport of Neisseria gonorrhoeae. The system consists of a flat plastic dish containing a culture medium and a well for a carbon dioxide-generating tablet. Specimens can be cultured on the Jembec plate at the patient’s bedside, incubated 18-24 hours, and then shipped to a laboratory. Another commercial system, the GonoPaK (Becton Dickinson Microbiology Systems, Cockeysville, MD) is a modification of the Jembec plate.

34
Q

Which of the following conditions would be the most detrimental to N. gonorrhoeae in a clinical specimen?

1 Collection on a calcium alginate swab containing charcoal and transported in Amies transport medium
2 Collection on a cotton swab and transported in a carbon dioxide-containing transport container
3 Collection on a calcium alginate swab and refrigerated until the specimen can be hand-carried to the laboratory for culture
4 Collection on rayon swab and held at room temperature
5 Collection on cotton swabs and inserted into Stuart’s transport medium at ambient temperature

A

Collection on a calcium alginate swab and refrigerated until the specimen can be hand-carried to the laboratory for culture

Both N. gonorrhoeae and N. meningitidis are extremely sensitive to storage at refrigerator temperature prior to culture. Regardless of the type of swab used, the specimen suspected of containing either of these organisms should be held and transported un-refrigerated, either at room temperature (ambient) or at 35° C.

35
Q

The Jembec plate and the GonoPak (Becton Dickinson Microbiology Systems, Cockeysville, Md.) are used to transport specimens suspected of containing Neisseria gonorrhoeae. The unique feature of these two commercially available systems is that they both

1 Use a special formulation of medium that does not contain blood
2 Use a CO2-generating tablet
3 Can be transported at refrigerator temperature without damaging the organism
4 Have media formulations that preclude the use of blood or carbon-dioxide
5 Include a built-in 35° C temperature generator

A

Use a CO2-generating tablet

Both the Jembec (John E. Martin Biological Environmental Chamber) and the GonoPac have a built-in carbon-dioxide generating tablet which is critical to the survival of N. gonorrhoeae during transport. The GonoPak is a modification of the Jembec plate.

36
Q

A positive oxidase test can be used to presumptively identify

1 Most any gram negative rod isolated from feces
2 Gram negative diplococci isolated from the urethra
3 Gram positive cocci isolated from the throat
4 Gram positive rods isolated from sputum
5 Gram negative coccobacilli isolated from endocervical smears

A

Gram negative diplococci isolated from the urethra

Neisseria gonorrhoeae are Gram negative diplococci isolated from the urethra and are oxidase positive. Gram negative rods from feces are likely to be members of the Enterobacteriaceae that are oxidase negative. Gram positive cocci from the throat are likely to be streptococci that are oxidase negative. Gram positive rods from sputum are likely to be contaminants as would Gram negative coccobacilli from the endocervix.

37
Q

Which of the following non-specific host defenses plays an important role in preventing the colonization of the stomach by pathogenic bacteria?

1 Ciliated epithelium
2 Competition by normal flora
3 Secretory IgA
4 Lymphoid follicles
5 Hydrochloric acid

A

Hydrochloric acid

In the stomach, it is primarily the presence of hydrochloric acid (HCl) and the production of mucous that are responsible for the prevention of colonization by pathogenic bacteria. Other factors involved include the mechanical barrier effect produced by sphincters, and the effects of flushing and peristalsis. Ciliated epithelium is not present in the stomach and, unlike other sites, competition by normal flora, secretory IgA, or the presence of lymphoid follicles do not play a role in the prevention of colonization by pathogenic bacteria.

38
Q

Ability of an organism to produce pathologic changes or disease in the host is known as

1 Virulence
2 Inflammation
3 Toxigenicity
4 Pathogenicity
5 Invasiveness

A

Pathogenicity

The capacity of pathogens to cause disease depends on their abilities to infect the host or protect themselves against the body’s defenses, to invade and multiply in tissues, and to cause damage to the tissue.
Virulence is the measure of degree of pathogenicity and can be defined as the sum of infectivity, invasiveness, and toxigenicity.
Inflammation is a nonspecific pathologic process consisting of a dynamic complex of cytologic and histologic reactions that occur in response to an injury or abnormal stimulation by a physical, chemical, or biologic agent.
Toxigenicity is the ability of the organism to produce toxins.
Invasiveness is the ability of the organism to penetrate and grow in the host tissue.

39
Q

Substances that stick to a microorganism’s surface and make it more attractive to phagocytic cells are known as

1 Interferons
2 Bacteriocins
3 Lysozymes
4 Opsonins
5 Phagosomes

A

Opsonins

Opsonins stick to the surface of a microorganism and enhance phagocytosis.
Interferons are the class of small, antiviral glycoproteins produced by cells infected with an animal virus.
Bacteriocins are proteins produced by certain bacteria that can kill other bacteria.
Lysozyme is a digestive enzyme found in lysozymes, tears, and other body fluids and is destructive to bacterial cell walls.
Phagosome is a membrane bound vesicle containing an ingested particle and is found in phagocytic cells.

40
Q

Endotoxins are

1 Mostly heat labile
2 Always highly toxic in microgram quantities
3 Known as lipopolysaccharides
4 Used to form toxoids
5 Produced by gram positive bacteria only

A

Known as lipopolysaccharides

Endotoxins are the lipopolysaccharide portion of the cell wall produced by gram negative bacteria only. They are heat stable and cannot be used to form toxoids. Endotoxins are toxic in milligram quantities, while most exotoxins are toxic in microgram quantities.
Endotoxin stimulates the fever center in the hypothalamus and causes hypotension. It initiates coagulation that can result in intravascular coagulation.

41
Q

The enzyme that enables an organism to clot plasma and form a fibrin coat is

1 Streptokinase
2 Hyaluronidase
3 Staphylokinase
4 Coagulase
5 Collagenase

A

Coagulase

Coagulase is the exoenzyme produced by Staphylococcus aureus, which causes citrated blood plasma to clot in the laboratory. However, kinases dissolve fibrin clots, thus enabling the organism to invade and spread throughout the body.
Hyaluronidase enables pathogen to spread through connective tissue by breaking down hyaluronic acid, the “cement” that holds tissue cells together.
Collagenase breaks down collagen, the supportive protein found in tendons.

42
Q

The ability of pathogens to invade, infect, cause damage, and produce disease in the host is termed

1 Pathogenicity
2 Toxigenicity
3 Virulence
4 Infection
5 Attenuation

A

Virulence

Virulence is the ability of a pathogen to invade, infect, cause damage, and produce disease in the host.
Pathogenicity is the ability of an organism to produce pathologic changes or disease in the host.
Toxigenicity is the ability of the organism to produce toxins.
Infection occurs when a pathogenic microbe is able to multiply in the tissues where it lodges.
Attenuation is the process by which the microorganism is rendered less pathogenic or avirulent.

43
Q

Dimorphic fungi

1 Are always non-pathogenic to humans
2 Can grow as yeast at 37° C and as mold at 23° C
3 Produces yeast forms at 23° C and motile rod forms at 45° C
4 Does not reproduce asexually

A

Can grow as yeast at 37° C and as mold at 23° C

Dimorphic fungi have the ability to grow in yeast form or mold form depending on the environmental conditions of growth. Most dimorphic fungi can grow as yeast at 30° C to 37° C or in tissue and as mold at lower temperatures (23° C to 25° C).
Sporothrix schenckii, Blastomyces dermatitidis, Coccidioides immitis, and Histoplasma capsulatum are dimorphic fungi that are highly pathogenic to humans. Dimorphic fungi reproduce asexually.

44
Q

White piedra is caused by Trichosporon beigelii. Hair submitted for isolation of Trichosporon beigelii should be cultured on:

1 Sabouraud dextrose agar with cycloheximide and chloramphenicol
2 Sabouraud dextrose agar with cycloheximide, but without chloramphenicol
3 Sabouraud dextrose agar with chloramphenicol, but without cycloheximide
4 Sabouraud dextrose agar without chloramphenicol and cycloheximide

A

Sabouraud dextrose agar with chloramphenicol, but without cycloheximide

Trichosporon beigelii causes white piedra characterized by soft, cream colored growth along the infected hair shafts. The hair shaft of axilla, scalp, beard, and genital areas are affected with this fungal infection. Trichosporon beigelii is sensitive to cycloheximide, therefore, the hair infected with Trichosporon beigelii should be cultured on Sabouraud dextrose agar with chloramphenicol, but without cycloheximide. Trichosporon beigelii produces cream color yeast like colonies on the culture media after 3-4 days of incubation. The colony turns yellowish gray on extended incubation (2-4 weeks). Microscopic examination of the slide culture reveals hyaline hyphae with many arthrospores and some blastospores.

45
Q

When delay of 18 to 20 hours is anticipated in the processing of a viral specimen, the specimen should be

1 Frozen at -20° C
2 Left in incubator at 37° C
3 Left at room temperature
4 Stored at 4° C

A

Stored at 4° C

It is preferred that the virus specimen is processed promptly as it enhances the viral detection. However, when delay of less than 24 hours is anticipated, the specimen for viral isolation should be stored at 4° C. Freezing of the specimen at -20° C can lead to loss of the infectivity of some of the virus. Storage at room temperature can also cause significant loss in infectious titer of viruses, such as influenza virus and varicella zoster virus.
The viral specimen should be snap-frozen at least at -70° C in case of prolonged delay (more than 24 hours). The viral specimen should be transported in dry ice or with liquid nitrogen.

46
Q

Most appropriate specimens for virus detection in cytomegalic inclusion disease in a newborn are

1 Urine or saliva
2 Stool or serum
3 Skin scraping and stool
4 Pericardial fluid and throat swab
5 Genital swab and plasma

A

Urine or saliva

Infection with cytomegalovirus during pregnancy can lead to congenital abnormalities in the newborn. The cytomegalic inclusion disease is a prenatal infection with cytomegalovirus. Cytomegalovirus can be isolated from the urine or saliva of a newborn with cytomegalic inclusion disease. The peripheral blood leukocyte is used for viral detection in case of viremia. Urine specimen is also used for the extraction of viral DNA in DNA hybridization test.

47
Q

An experiment is devised to study the in vitro effects of cytokines and other chemokines on macrophages. Recombinant interferons alpha, beta, and gamma, interleukin-1, interleukin-2, interleukin-4, interleukin-6, granulocyte macrophage colony stimulating factor and tumor growth factor beta were added individually to macrophages. Which of the following cytokines or chemokines activate macrophages by a mechanism that involves increased ion exchange?

1 Interferon-alpha, interferon-beta, and interferon-gamma
2 Interleukin-1
3 Granulocyte macrophage colony stimulating factor
4 Tumor growth factor-beta
5 Interferon-beta
6 interferon-gamma
7 interleukin-2
8 interleukin-4
9 interleukin-6

A

interferon-gamma

Increased antimicrobial and antitumor activity of macrophages results from activation by many types of cytokines. Inflammatory proteins, interleukins, interferon, and growth factors all regulate the function of macrophages. Interferon ã appears to increase Na+/H+ exchange in macrophages.

48
Q

Your patient is a 55-year-old woman with a fever of 40° C and a very painful cellulitis of the left buttock. The skin appears necrotic and there are several fluid-filled bullae. Crepitus can be felt, indicating gas in the tissue. A Gram stain of the exudate reveals large gram-positive rods. What is the mechanism of action of the exotoxin of Clostridium perfringens (alpha toxin)?

1 Inactivates EF-2 by ADP-ribosylation
2 Blocks release of the inhibitory neurotransmitter glycine
3 It is a lecithinase
4 Stimulates adenylate cyclase by ADP-ribosylation
5 Blocks release of acetylcholine

A

It is a lecithinase

Multiple toxins are produced by Clostridium perfringens and other species of clostridia that cause gas gangrene. A total of seven lethal factors and five enzymes have been characterized, but no species of Clostridium makes all 12 products. The best characterized is the alpha-toxin, which is a lecithinase that hydrolyzes lecithin in the cell membrane, resulting in widespread cell death. The other four enzymes are collagenenases, protease, hyaluronidase and deoxyribonuclease. Certain strains of C. perfringens produce an enterotoxin that causes watery diarrhea. This enterotoxin acts as a superantigen.
Tetanus toxin, produced by Clostridium tetani, is a neurotoxin that prevents release of the inhibitory neurotransmitter glycine. When the inhibitory neurons are nonfunctional, the excitatory neurons are unopposed, leading to muscle spasms and a spastic paralysis.
Botulinum toxin, produced by Clostridium botulinum, is a neurotoxin that blocks the release of acetylcholine at the synapse, producing a flaccid paralysis.
Diphtheria toxin ADP- ribosylates elongation factor-2, thereby inactivating it and resulting in the inhibition of protein synthesis.
Vibrio cholerae and E. coli produce exotoxins that ADP-ribosylate Gs protein, thereby inactivating it. This causes an increase in adenylate cyclase activity, a consequent increase in the amount of cyclic AMP, and the production of watery diarrhea. Bordetella pertussis ADP-ribosylates Gi protein and inactivates it. This turns on adenylate cyclase, causing an increase in the amount of cAMP, which plays a role in causing the symptoms of whooping cough.

49
Q

Four days after major abdominal surgery, a 60-year-old woman suddenly develops severe abdominal pain and fever (39.2° C). Her vital signs were BP 90/60 mmHg, pulse 110/min. Examination of fluid removed from her peritoneum shows abundant leukocytes and many gram-negative rods and gram-positive cocci. Which one of the following best describes the mode of action of endotoxin?

1 Degrades lecithin in cell membranes
2 Inactivates elongation factor 2
3 Blocks release of neurotransmitter glycine
4 Causes the release of tumor necrosis factor
5 Is cytotoxic

A

Causes the release of tumor necrosis factor

Endotoxins are integral parts of both gram-negative rods and cocci, in contrast to exotoxins, which are released from the cell. All endotoxins produce the same generalized effects of fever and shock. Endotoxins are lipopolysaccharides. The toxic portion of the molecule is lipid A, which is composed of disaccharides with several fatty acids attached. Beta-hydroxymyristic acid is always one of the fatty acids. The biologic effects of endotoxin include fever; hypotension, shock and impaired perfusion of essential organs owing to bradykinin-induced vasodilatation, increased vascular permeability and decreased peripheral resistance; disseminated intravascular coagulation due to activation of the coagulation system through Hageman factor, resulting in thrombosis, a petechial or purpuric rash and tissue ischemia; activation of the alternative pathway of the complement cascade, resulting in inflammation and tissue damage; an activation of macrophages and many clones of B lymphocytes. Endotoxins do not cause these effects directly. Rather, they elicit the production of host factors such as interleukin-1 and cachectin (tumor necrosis factor).
Tetanus toxin, produced by Clostridium tetani, is a neurotoxin that prevents release of the inhibitory neurotransmitter glycine.
Two exotoxins are produced by Clostridioides difficile, both of which are involved in the pathogenesis of pseudomembranous colitis. Exotoxin A is an enterotoxin that causes watery diarrhea. Exotoxin B is a cytotoxin that damages the colonic mucosa and causes pseudomembranes to form.
Diphtheria toxin ADP- ribosylates elongation factor-2, thereby inactivating it and resulting in the inhibition of protein synthesis.
The best-characterized toxin of Clostridium perfringens, the alpha-toxin, which is a lecithinase, hydrolyzes lecithin in the cell membrane, resulting in widespread cell death.

50
Q

A man brings his uncle to your office and tells you that the uncle is in the United States on vacation and lives in the Caribbean. He complains of a very sore throat and has problems swallowing. The lymph glands in his neck are very swollen. You suspect that he has diphtheria, which is confirmed by a throat culture. You know that diphtheria toxin is a powerful inhibitor of protein synthesis, which functions by

1 catalyzing the ADP-ribosylation of an elongation factor
2 dissociating the proteins of the large ribosomal subunit
3 competing with methionyl-tRNA for binding to the initiation codon
4 cleaving the 28S ribosomal RNA
5 binding irreversibly to an elongation factor

A

catalyzing the ADP-ribosylation of an elongation factor

Diphtheria develops in the throat. The first symptoms are a sore throat and mild fever. A membrane that forms over the throat and tonsils can make it hard to swallow. The infection also causes the lymph glands and tissue on both sides of the neck to swell to an unusually large size. Diphtheria toxin (produced by Corynebacterium diphtheriae) is a multi-domain protein toxin composed of a receptor-binding domain, a membrane insertion domain, and a catalytic domain. It enters the cell by a receptor-mediated mechanism, whereupon it is cleaved into a 21 kDa A fragment and a 40-kDa B fragment. The B fragment contains the membrane insertion domain that allows the A fragment to enter the cell cytoplasm. This A fragment contains the catalytic domain that catalyzes the following ADP-ribosylation reaction:
EF2active + NAD+ D ADP-ribosyl-EF2inactive + nicotinamide + H+
The inactivation of elongation factor 2 (EF2), which catalyzes translocation in eukaryotic protein synthesis, leads to the inhibition of protein translocation and translation stops. A single molecule of the A fragment of diphtheria toxin can kill the cell by completely inactivating all EF2 in the cell. The toxin does not function by binding to the EF, but by chemically modifying the EF.

51
Q

A man brings his uncle to your office and tells you that the uncle lives in the Caribbean but is in the United States on vacation. He complains of a very sore throat and has problems swallowing. The lymph glands in his neck are very swollen. You suspect that he has diphtheria, which is confirmed by a throat culture. One domain of the toxin is involved in the inhibition of protein synthesis. A second domain is involved in

1 toxin binding to the cell receptor
2 inhibiting DNA replication
3 cleavage of the 28S ribosomal RNA
4 binding and sequestering elongation factors
5 transporting toxin into the nucleus

A

toxin binding to the cell receptor

Diphtheria develops in the throat. The first symptoms are a sore throat and mild fever. A membrane that forms over the throat and tonsils can make it hard to swallow. The infection also causes the lymph glands and tissue on both sides of the neck to swell to an unusually large size. Diphtheria toxin (produced by Corynebacterium diphtheriae) is a multi-domain protein toxin composed of a receptor-binding domain, a membrane insertion domain, and a catalytic domain. It enters the cell by a receptor-mediated mechanism, whereupon it is cleaved into a 21 kDa A fragment and a 40-kDa B fragment. The B fragment contains the membrane insertion domain that allows the A fragment to enter the cell cytoplasm. This A fragment contains the catalytic domain that catalyzes the following ADP-ribosylation reaction:
EF2active + NAD+ D ADP-ribosyl-EF2inactive + nicotinamide + H+
The inactivation of elongation factor 2 (EF2), which catalyzes translocation in eukaryotic protein synthesis, leads to the inhibition of protein translocation and translation stops. A single molecule of the A fragment of diphtheria toxin can kill the cell by completely inactivating all EF2 in the cell. The toxin does not function by binding to the EF, but by chemically modifying the EF.

52
Q

Case
A 20-year-old woman presents with a malodorous vaginal discharge. She has had this discharge for approximately 2 weeks. She has been sexually inactive for the past year. She does not note any itching or abdominal/pelvic pain. Physical examination reveals a homogeneous gray discharge. There appears to be no redness or ulceration of the vulva and surrounding area. Some of the vaginal discharge is obtained and mixed with 10% KOH, producing a fishy amine odor. A Gram stain of the vaginal discharge reveals diagnostic “clue” cells. Refer to the image.

Based on the clinical presentation and test results, what is the most likely diagnosis?

1 Gonorrhea cervicitis
2 Bacterial vaginosis
3 Candidiasis/vulvovaginitis
4 Chlamydial endocervicitis
5 Trichomoniasis

A

Bacterial vaginosis

Bacterial vaginosis (nonspecific vaginitis) is a disease that is characterized by a malodorous vaginal discharge. The diagnosis of bacterial vaginosis is based on the presence of at least 3 of the 4 following signs:

characteristic homogeneous gray discharge
presence of “clue” cells
vaginal pH greater than 4.5
the release of a fishy amine odor from vaginal secretions mixed with 10% potassium hydroxide (KOH)

Gonorrhea cervicitis is caused by Neisseria gonorrhoeae and is a common sexually transmitted disease. Clinical manifestations include vaginal discharge, dysuria, intermenstrual bleeding, purulent or mucopurulent endocervical discharge, and erythema. Many women have mild or asymptomatic infections. The organism is a gram-negative diplococcus that is oxidase positive, glucose positive, sucrose negative, ONPG negative, and nitrate negative. When a gram-negative diplococcus is detected in the Gram-stained smear of discharge material, it is not diagnostic for a woman of Neisseria gonorrhoeae because there are other normal gram-negative diplococci present in the vaginal tract that are part of the normal flora. DNA probes, culture with biochemical identification, and EIA methods are used to detect and identify the organism.
Candidiasis/vulvovaginitis is often a chronic disease and is the most common candida infection. Candida albicans is the most common Candida species causing candidiasis. The disease causes an intense vaginal itching and burning sensation that is accompanied by a pruritus. There is usually a discharge that is described as thick and curd-like. The vulva is inflamed, and ulcerations can occur. Gram stains of the discharge will typically contain large amounts of yeast cells.
Chlamydial endocervicitis is a mucopurulent infection caused by Chlamydia trachomatis, a sexually transmitted disease. The organism cannot be detected by normal Gram staining methods and requires fluorescent antibody staining techniques, cell culture, DNA probes, or EIA methods to detect it. It is the most common sexually transmitted pathogen. The organism also causes urethritis, endometritis, salpingitis, tubal factor infertility, and ectopic pregnancy.
Trichomoniasis is caused by Trichomonas vaginalis, a flagellated protozoan. It is characterized by vaginal itching and discomfort with a discharge that is often copious, frothy, and malodorous. Person-to-person spread is primarily through sexual contact. There is severe itching of the vulva and inner thighs. Definitive diagnosis is made by demonstrating the motile organisms in fresh secretions; it can also be demonstrated in spun down urine during a urinalysis.

53
Q

An elderly patient hospitalized with pneumonia is being treated with clindamycin. He develops diarrhea and also complains of abdominal cramps. Stool examination shows the presence of a toxin that kills cultured epithelial cells. A stool culture reports an anaerobic gram-positive rod. The same organism is cultured from his bedpan. Which of the following is most likely to sterilize the bedpan?

1 Washing with alcohol and drying in the sun
2 Exposure to ethyl alcohol for 1 hour
3 Exposure to benzalkonium chloride for 1hour
4 Boiling for 45 minutes
5 Exposure to saturated steam (121° C) for 15 minutes
6 Heating in an oven at 150° C for 30 minutes

A

Exposure to saturated steam (121° C) for 15 minutes

This is a case of pseudomembranous colitis caused by Clostridioides difficile.
C. Difficile is a spore-forming anaerobic gram-positive rod. It is an important cause of nosocomial disease. It is transmitted by the fecal-oral route. Exotoxin B can be detected by its cytopathic effect on cells in tissue culture or by ELISA.
Sterilization of the bedpan would require a process that can kill spores. Among the listed options, sterilization with wet heat at 121° C for at least 15 minutes is the only process that would achieve sterilization of the bedpan.
Treatment involves discontinuation of suspected antibiotics. Oral metronidazole and vancomycin may prove effective.

54
Q

Case
A 20-year-old woman has acute onset diarrhea. She reports no previous medical problems. Acid-fast stain of stool is positive.

What is the most likely diagnosis?

1 Candida
2 Histoplasma capsulatum
3 Nocardia
4 Cryptococcus neoformans
5 Actinomyces
6 Blastomyces dermatitidis
7 Aspergillus
8 Cryptosporidium
9 Babesia microti
10 Mycobacterium avium

A

Cryptosporidium

Cryptosporidium causes diarrhea. The best diagnosis is sugar flotation and stool for acid-fast stain. Fever and hemolysis are characteristic for protozoan disease caused by Babesia microti. Transmission occurs by tick, and peripheral blood smear is used for diagnosis.
Actinomyces inhabits the mouth, gut, and vagina, and produces cervicofacial osteomyelitis and abscess, pneumonia with empyema, and intra-abdominal and pelvic abscess, generally associated with intrauterine contraceptive devices.
Candida species are oval yeast that often colonizes the mouth, gastrointestinal tract, and vagina of healthy individuals. They may produce disease by overgrowth and/or invasion.
Candida stomatitis (thrush) often occurs in individuals who are receiving antibiotic or corticosteroid therapy or who have impairments of cell-mediated immunity.
Vulvovaginitis due to Candida may occur in this same setting but is also seen among women with diabetes mellitus or with no apparent predisposing factors.
Mycobacterium avium has been implicated as an important cause of both local (pulmonary) and disseminated infections among patients with the acquired immunodeficiency syndrome. Blood cultures are often positive. Mycobacteria are typically found in bone marrow, lymph nodes, and liver.
Cryptococcus neoformans is a yeast with a large polysaccharide capsules. It produces a self-limited or chronic pneumonia, but the most common clinical manifestation of infection with this fungus is chronic meningitis. Although patients with impairment in cell-mediated immunity are at risk for cryptococcal meningitis, some patients with this syndrome have no identifiable immunodeficiency.

55
Q

A 45-year-old woman comes into your office because of abdominal cramps and bloody diarrhea. She has just returned from her vacation in India, where she ate many of the local foods. BP 120/80 mm Hg, P 84, T 37,8° C. You make a stain for fecal leukocytes, which reveals many PMNs. From the stool culture a gram-negative rod was isolated that forms colorless colonies on eosin-methylene blue (EMB) agar. Triple sugar iron agar shows an alkaline slant, acid butt, no gas and no H2S. The urease test was negative. Which one of the following organisms is the most likely cause?

1 Shigella dysenteriae
2 Escherichia coli
3 Vibrio cholerae
4 Vibrio parahaemolyticus
5 Yersinia enterocolitica

A

Shigella dysenteriae

Specimens suspected of containing members of the Enterobacteriaceae and related organisms are usually inoculated onto two media, a blood agar plate and a selective differential medium such as MacConkey’s agar or eosin-methylene blue (EMB) agar. Lactose fermentation is the most important metabolic criterion used in the identification of these organisms. On these media, Salmonella and Shigella - the nonlactose fermenters form colorless colonies, whereas the lactose fermenters form colored colonies.
Additional screening tests, triple sugar iron (TSI) agar and urea agar, are done prior to the definitive identification procedures.

Triple sugar iron agar reactions

Reactions

Slant
Butt
Gas
H2S
Representative Genera

Acid
Acid
+
-
Escherichia, Enterobacter, Klebsiella

Alkaline
Acid
-
-
Shigella, Serratia

Alkaline
Acid
+
+
Salmonella*, Proteus

Alkaline
Alkaline
-
-
Pseudomonas

*Salmonella typhi is the major exception - it does not form gas, and produces only a small amount of H2S.
The important organisms that are urease-positive are Proteus species and Klebsiella pneumoniae.

56
Q

A 39-year-old woman comes into your office because she has had severe diarrhea for the past 2 days, accompanied by fever and a shaking chill. She uses antacids. Otherwise she is healthy. A stain for fecal leukocytes reveals many PMNs. Test for occult blood was positive. From the stool culture a gram-negative rod was isolated that does not ferment lactose. Triple sugar iron agar shows an alkaline slant, acid butt, with both gas and H2S. The urease test was negative. Which one of the following organisms is the most likely cause?

1 Salmonella typhimurium
2 Escherichia coli
3 Campylobacter jejuni
4 Vibrio parahaemolyticus
5 Vibrio cholerae

A

Salmonella typhimurium

Specimens suspected of containing members of the Enterobacteriaceae and related organisms are usually inoculated onto two media, a blood agar plate and a selective differential medium such as MacConkey’s agar or eosin-methylene blue (EMB) agar. Lactose fermentation is the most important metabolic criterion used in the identification of these organisms. On these media, Salmonella and Shigella - the nonlactose fermenters form colorless colonies, whereas the lactose fermenters form colored colonies.
Additional screening tests, triple sugar iron (TSI) agar and urea agar, are done prior to the definitive identification procedures.

Triple sugar iron agar reactions

Reactions

Slant
Butt
Gas
H2S
Representative Genera

Acid
Acid
+
-
Escherichia, Enterobacter, Klebsiella

Alkaline
Acid
-
-
Shigella, Serratia

Alkaline
Acid
+
+
Salmonella*, Proteus

Alkaline
Alkaline
-
-
Pseudomonas

*Salmonella typhi is the major exception - it does not form gas, and produces only a small amount of H2S.
The important organisms that are urease-positive are Proteus species and Klebsiella pneumoniae.

57
Q

An elderly woman was admitted to the hospital for a septic wound treatment on her lower abdomen which developed after a colon operation. Her wound was 1.5-cm open, and oozing a yellow exudate. Which is the best way of sterilizing the sheet of the bed contaminated with the septic wound’s secretion?

1 Using anionic agent solvents
2 Using nonionic agent solvents
3 Dry heat sterilization
4 Autoclave sterilization
5 Radiation

A

Autoclave sterilization

Sterilization is the complete inactivation of all forms of microbial life including the spores.
Antiseptics and disinfectants either inhibit or kill the microorganisms but do not kill the bacterial spores (disinfection). Anionic, non-anionic, and cationic agents (surface active agents) disrupt the lipid bilayer by altering the orderly arrangement of proteins and lipids of membrane. They are used for domestic cleanings as disinfectants.
Heat inactivates microorganisms by oxidizing intracellular components. Heat also denatures proteins by coagulation and causes breaks in nucleic acids. Moist and dry heats are very effective means of sterilizing materials.
Moist heat, steam sterilization is carried out in an autoclave in the presence of saturated steam under pressure for 15 minutes at 121° C. Materials like linen, which is suitable for steam, is best sterilized with autoclave sterilization.
Dry heat is less efficient than moist heat, therefore higher temperatures are used for a longer periods. Nevertheless, dry heat is more suitable for glasses, surgical equipment, mineral oils, waxes, powders, and glasses.
Pasteurization is a process using mild heat to reduce or destroy microorganisms to a set level from food.
Radiation is an electromagnetic energy of a specific length, which can have lethal effects on microorganisms. The two types used are UV light and X-rays. The use of UV light is limited because of its damage to the cornea and skin. X-rays kill vegetative cells but spores are resistant to radiations owing to lower water content.

58
Q

Cholera toxin is one of a family of bacterial toxins that exert their physiologic effect by altering an intracellular signal transduction pathway. Cholera toxin causes a single covalent modification that eventually leads to an increase in the intracellular level of

1 cAMP
2 Calcium
3 GTP
4 cGMP
5 Phosphotyrosine

A

cAMP

Signal transduction pathways are a mechanism by which an external signal (chemical, mechanical, or electrical) is amplified and converted to a cellular response. For example, the binding of a hormone to its cell surface receptor generates various signals inside the cell that can result in changes in many aspects of cell metabolism. Compounds other than hormones, including neurotransmitters and ions, can generate signals. One major signaling pathway in the cell that converts an external signal to an intracellular message involves the activation of G proteins. After a ligand binds to its membrane receptor, a G-protein transducer is activated by the exchange of bound GDP for GTP. This activated G protein then interacts with other proteins to generate a second messenger within the cell. G-protein activation of membrane-bound adenylate cyclase produces the second messenger cAMP. Cholera toxin is the major virulence factor secreted by Vibrio cholerae. The A1 fragment of cholera toxin catalyzes the transfer of ADP-ribose to the adenylate cyclase regulatory protein Gs. This single covalent modification (ADP-ribosylation) results in the intracellular accumulation of cAMP, triggering a signaling cascade, which eventually leads to the massive diarrhea that characterizes cholera.

59
Q

Cholera toxin is one of a family of bacterial toxins that exert their physiologic effect by altering an intracellular signal transduction pathway. The specific reaction that is catalyzed by a subunit of cholera toxin is

1 Hydrolysis of peptide bonds
2 Deamination of amino groups
3 Myristoylation of a glycine
4 Phosphorylation of tyrosine
5 ADP-ribosylation

A

ADP-ribosylation

Signal transduction pathways are a mechanism by which an external signal (chemical, mechanical, or electrical) is amplified and converted to a cellular response. For example, the binding of a hormone to its cell surface receptor generates various signals inside the cell that can result in changes in many aspects of cell metabolism. Compounds other than hormones, including neurotransmitters and ions, can generate signals. One major signaling pathway in the cell that converts an external signal to an intracellular message involves the activation of G proteins. After a ligand binds to its membrane receptor, a G-protein transducer is activated by the exchange of bound GDP for GTP. This activated G protein then interacts with other proteins to generate a second messenger within the cell. G-protein activation of membrane-bound adenylate cyclase produces the second messenger cAMP. Cholera toxin is the major virulence factor secreted by Vibrio cholerae. The A1 fragment of cholera toxin catalyzes the transfer of ADP-ribose to the adenylate cyclase regulatory protein Gs. This single covalent modification (ADP-ribosylation) results in the intracellular accumulation of cAMP, triggering a signaling cascade, which eventually leads to the massive diarrhea that characterizes cholera.

60
Q

Since macromolecular antigens contain several epitopes, some of these macromolecules can be altered without totally changing the immunogenic and antigenic structure of the entire molecule. This concept is important in relation to immunization against highly pathogenic microorganisms or highly toxic compounds. It is possible to destroy the biologic activity of a broad variety of toxins without appreciably affecting their antigenicity or immunogenicity. What are toxins that have been modified to the extent that they are no longer toxic but still maintain some of their immunochemical characteristics called?

1 RTX toxins
2 Immunotoxins
3 Immunophilins
4 Endotoxins
5 Toxoids

A

Toxoids

Modified toxin molecules, or toxoids, lack toxic activity, but they cross-react immunologically with the toxin. Accordingly, it is possible to immunize individuals with the toxoid and thereby induce immune responses to some of the epitopes that the toxoid still shares with the native toxin because these epitopes have not been destroyed by the modification. For example, diphtheria and tetanus toxins are among the bacterial toxins in which the toxic and the receptor-binding functions of the molecule are on 2 separate chains. Individuals can be immunized with modified toxin molecules in which the toxic chain has been denatured. These toxoid molecules lack toxic activity but retain the receptor-binding site, so that immunization with the toxoid induces neutralizing antibodies effective in protecting against the native toxin.
One group of toxins has been designated RTX toxins (repeats in toxin) by virtue of a common structural feature: the presence of an array of a 9 amino acid repeat to which calcium binds, thereby activating the toxins, which form membrane pores of varying sizes. They constitute the largest group of bacterial pore-forming toxins and are widespread among Gram-negative pathogens.
Linking a tumor-specific or tumor-selective monoclonal antibody to a toxin is a way to create an immunotoxin, a reagent that can be used to destroy all or part of a tumor. When the labeled antibody is internalized, the toxin is cleaved from the antibody in the cell’s endocytic compartment, allowing the toxin to penetrate and kill the cell.
The immunophilins are a family of intracellular proteins that bind to certain compounds, forming complexes that interfere with signaling pathways important for the clonal expansion of lymphocytes.
Endotoxins are part of the outer membrane of Gram-negative bacteria. Some of the diseases in which endotoxins may play a part are typhoid fever, tularemia, plague and brucellosis, and a variety of hospital-acquired infections caused by opportunistic Gram-negative pathogens. Endotoxin is a complex lipopolysaccharide that displays a large array of biological effects.

61
Q

For some bacterial infections, the manifestations of disease are caused by toxins. Bacterial toxins are proteins that produce their physiologic effects at minute concentration. The interaction of certain toxins with the immune system can have major immunologic consequences when they are able to bind the T cell receptor of large numbers of T cells. What are these toxins known as?

1 Immunotoxins
2 Superantigens
3 Immunophilins
4 Toxoids
5 RTX toxins

A

Superantigens

Superantigens are a distinct class of antigens that have a special mode of binding that enables them to stimulate very large numbers of T cells. They can bind to the MHC class II molecules outside the antigen-binding groove. This mode of stimulation does not prime an adaptive immune response specific for the pathogen. Instead it causes a massive production of cytokines by CD4 T cells. These cytokines have 2 effects on the host: systemic toxicity and suppression of the adaptive immune response. Both of these effects contribute to microbial pathogenicity. Superantigens are virulence factors for Staphylococcus aureus and Streptococcus pyogenes.
Linking a tumor-specific or tumor-selective monoclonal antibody to a toxin is a way to create an immunotoxin, a reagent that can by used to destroy all or part of a tumor. When the labeled antibody is internalized, the toxin is cleaved from the antibody in the cell’s endocytic compartment, allowing the toxin to penetrate and kill the cell.
The immunophilins are a family of intracellular proteins that bind to certain compounds, forming complexes that interfere with signaling pathways important for the clonal expansion of lymphocytes.
Diphtheria and tetanus toxins are among the bacterial toxins in which the toxic and the receptor-binding functions of the molecule are on 2 separate chains. Toxoids are modified toxin molecules that lack toxic activity but retain the receptor-binding site, so that immunization with the toxoid induces neutralizing antibodies effective in protecting against the native toxin.
One group of toxins has been designated RTX toxins (repeats in toxin) by virtue of a common structural feature: the presence of an array of a 9 amino acid repeat to which calcium binds, thereby activating the toxins that form membrane pores of varying sizes. They constitute the largest group of bacterial pore-forming toxins and are widespread among gram-negative pathogens.

62
Q

In infectious diseases, there is nearly always a certain amount of direct microbial damage to host tissues. Although a single molecule of some toxins is enough to kill a cell, other toxins do no more than impair cell function when present in sublethal concentrations. At low concentrations bacterial toxins can be potent inducers of what?

1 Immunotoxins
2 Immunophilins
3 Toxoids
4 Streptolysins
5 Cytokines

A

Cytokines

The immune system is regulated by soluble mediators called cytokines. These low-molecular-weight proteins are produced by virtually all cells of the innate and adaptive immune systems and in particular by T cells, which orchestrate many effector mechanisms. Their major functional activities are concerned with the regulation of the development and behavior of immune effector cells. Cytokines serve as chemical messengers within the immune system. Some bacterial toxins induce cytokines. For example, the pneumolysin of Streptococcus pneumoniae and anthrax lethal toxin make monocytes release IL-1 and TNF-α.
Linking a tumor-specific or tumor-selective monoclonal antibody to a toxin is a way to create an immunotoxin, a reagent that can by used to destroy all or part of a tumor. When the labeled antibody is internalized, the toxin is cleaved from the antibody in the cell’s endocytic compartment, allowing the toxin to penetrate and kill the cell.
The immunophilins are a family of intracellular proteins that bind to certain compounds forming complexes that interfere with signaling pathways important for the clonal expansion of lymphocytes.
Diphtheria and tetanus toxins are among the bacterial toxins in which the toxic and the receptor-binding functions of the molecule are on 2 separate chains. Toxoids are modified toxin molecules that lack toxic activity but retain the receptor-binding site, so that immunization with the toxoid induces neutralizing antibodies effective in protecting against the native toxin.
As they multiply in tissues, some bacteria release soluble materials that are lethal for phagocytes. Pathogenic streptococci release hemolysins or streptolysins that lyse red blood cells and have an important toxic action on polymorphs and macrophages.

63
Q

Anthrax is an infectious disease caused by Bacillus anthracis. Infection takes place following the ingestion or inhalation of spores, or by the entry of spores through abraded skin. The anthrax toxin complex consists of 3 components, none of which are toxic alone. What best classifies the toxin components that cause anthrax?

1 Binary toxins
2 RTX toxins
3 Immunotoxins
4 Immunophilins
5 Endotoxins

A

Binary toxins

Protein toxins are either secreted by Gram-positive and Gram-negative bacteria or released upon lysis from these microbes. Some of them are enzymes. Binary toxins are comprised of 2 proteins, only 1 of which is toxic. The other, however, is necessary at some stage for manifestation of toxicity. Anthrax toxin is an example, and others include Serratia and Proteus cytolysins, staphylococcal leukocidins, and the binary toxins found in some strains of Clostridioides difficile.
One group of toxins has been designated RTX toxins (repeats in toxin) by virtue of a common structural feature - the presence of an array of a 9 amino acid repeat which calcium binds to, activating the toxins, which then form membrane pores of various sizes. RTX toxins constitute the largest group of bacterial pore-forming toxins and are widespread among Gram-negative pathogens.
Linking a tumor-specific or tumor-selective monoclonal antibody to a toxin is one way to create an immunotoxin, a reagent that can by used to destroy all or part of a tumor. When the labeled antibody is internalized, the toxin is cleaved from the antibody in the cell’s endocytic compartment, which allows the toxin to penetrate and kill the cell.
Immunophilins are a family of intracellular proteins that bind to certain compounds forming complexes that interfere with signaling pathways important for the clonal expansion of lymphocytes.
Endotoxins are part of the outer membrane of Gram-negative bacteria. Some of the diseases in which endotoxins play a part include typhoid fever, tularemia, bubonic plague, brucellosis, and a variety of hospital-acquired infections caused by opportunistic Gram-negative pathogens. Endotoxins are complex lipopolysaccharides that display a large array of biological effects.

64
Q

Anthrax is caused by infection with Bacillus anthracis. In humans, anthrax infection is fairly rare; the risk of infection is about one in 100,000. Anthrax spores germinate inside macrophages and then the bacteria form substances that kill macrophages, increase vascular permeability, and give rise to local edema and hemorrhage. The anthrax bacterium produces components that act as what?

1 Immunotoxins
2 Immunophilins
3 Toxoids
4 RTX toxins
5 Binary toxins

A

Binary toxins

In addition to a cell-wall capsule, virulent strains of Bacillus anthracis produce three distinct components called the anthrax toxin. Each component is a thermolabile protein with a molecular weight of approximately 80 kDa. Factor I is called the edema factor (EF), and causes the widespread edema seen in the disease. Factor II is the protective factor (PF), because it induces protective antitoxic antibodies in guinea pigs. Factor III is known as the lethal factor (LF) because it is essential for the lethal effects of the anthrax toxin. None of the factors is toxic by itself, but when combined as binary toxins they can be lethal. Experiments show that the anthrax toxin has the familiar A-B enzymatic binding structure of bacterial exotoxins with PF acting as the B fragment and either EF or LF acting as the active A fragment.
Linking a tumor-specific or tumor-selective monoclonal antibody to a toxin is a way to create an immunotoxin, a reagent that can be used to destroy all or part of a tumor. When the labeled antibody is internalized, the toxin is cleaved from the antibody in the cell’s endocytic compartment, allowing the toxin to penetrate and kill the cell.
The immunophilins are a family of intracellular proteins that bind to certain compounds forming complexes that interfere with signaling pathways important for the clonal expansion of lymphocytes.
Diphtheria and tetanus toxins are among the bacterial toxins in which the toxic and the receptor-binding functions of the molecule are on two separate chains. Toxoids are modified toxin molecules that lack toxic activity but retain the receptor-binding site, so that immunization with the toxoid induces neutralizing antibodies effective in protecting against the native toxin.
One group of toxins has been designated RTX toxins (repeats in toxin) by virtue of a common structural feature - the presence of an array of a nine amino acid repeat to which calcium binds thereby activating the toxins which form membrane pores of varying sizes. They constitute the largest group of bacterial pore-forming toxins and are widespread among Gram-negative pathogens.

65
Q

Staphylococcal bacteria cause a wide variety of human disorders, from food poisoning to rashes. Some strains of staphylococci cause toxic shock syndrome (TSS), a multisystem disease. The cause of TSS is a toxin called toxic shock syndrome toxin 1 (TSST-1), originally recognized as one of the pyrogenic staphylococcal enterotoxins. It is one of a group of toxins known as what?

1 Superantigens
2 Immunotoxins
3 Immunophilins
4 Toxoids
5 RTX toxins

A

Superantigens

Staphylococcal TSST-1 is known to be a superantigen. Superantigens are a distinct class of antigens that have a special mode of binding that enables them to stimulate very large numbers of T cells. They can bind to the MHC class II molecules outside the antigen-binding groove. This mode of stimulation does not prime an adaptive immune response specific for the pathogen. Instead it causes a massive production of cytokines by CD4 T cells. These cytokines have 2 effects on the host: systemic toxicity and suppression of the adaptive immune response. Both of these effects contribute to microbial pathogenicity.
Linking a tumor-specific or tumor-selective monoclonal antibody to a toxin is a way to create an immunotoxin, a reagent that can by used to destroy all or part of a tumor. When the labeled antibody is internalized, the toxin is cleaved from the antibody in the cell’s endocytic compartment, allowing the toxin to penetrate and kill the cell.
The immunophilins are a family of intracellular proteins that bind to certain compounds forming complexes that interfere with signaling pathways important for the clonal expansion of lymphocytes.
Diphtheria and tetanus toxins are among the bacterial toxins in which the toxic and the receptor-binding functions of the molecule are on 2 separate chains. Toxoids are modified toxin molecules that lack toxic activity but retain the receptor-binding site, so that immunization with the toxoid induces neutralizing antibodies effective in protecting against the native toxin.
One group of toxins has been designated RTX toxins (repeats in toxin) by virtue of a common structural feature: the presence of an array of a 9 amino acid repeat to which calcium binds, thereby activating the toxins which form membrane pores of varying sizes. They constitute the largest group of bacterial pore-forming toxins and are widespread among Gram-negative pathogens.

66
Q

Clostridium perfringens possess various virulence factors that enable it to break down tissue materials, thereby facilitating spread of the infection along tissue planes. Most of these factors are toxic to host cells and tissues, but C. perfringens α-toxin is easily the most important one. This is an example of a group of toxins that are what?

1 Binary toxins
2 RTX toxins
3 Phospholipases
4 Immunotoxins
5 Immunophilins

A

Phospholipases

Toxins are significant virulence factors for bacteria. The table lists some bacteria that produce toxins and the diseases they cause (refer to the table).
Disease Organism Toxin
Listeriosis Listeria monocytogenes Listeriolysin
Pneumonia Streptococcus pneumoniae Pneumolysin
Tetanus Clostridium tetani Tetanus toxin
Diphtheria Corynebacterium diphtheriae Diphtheria toxin
Botulism Clostridium botulinum Botulinum toxin
Gas gangrene Clostridium perfringens Clostridial toxin
Many bacterial enzymes are phospholipases. The best example is the α-toxin of C. perfringens, the Gram positive organism most commonly associated with gas gangrene. The microbe is strictly anaerobic, and its spores are ubiquitous in soil, dust, and air. The toxin is dermonecrotic, hemolytic, causes turbidity in lipoprotein-rich solutions, and is lethal. It was the first bacterial toxin to be characterized as an enzyme; it is a zinc metallophospholipase C that removes the head group, phosphoryl choline, from phosphatidyl choline and from sphingomyelin. It is of undoubted importance in gas gangrene.
Binary toxins are comprised of 2 proteins, only 1 of which is toxic, but the other is necessary at some stage for manifestation of toxicity. Anthrax toxin is one example, and two more are Serratia and Proteus cytolysins and staphylococcal leukocidins.
One group of toxins has been designated RTX toxins (repeats in toxin) by virtue of a common structural feature: the presence of an array of a 9 amino acid repeat to which calcium binds, thereby activating the toxins, which form membrane pores of varying sizes. They constitute the largest group of bacterial pore-forming toxins and are wdespread among gram-negative pathogens.
Linking a tumor-specific or tumor-selective monoclonal antibody to a toxin is a way to create an immunotoxin, a reagent that can be used to destroy all or part of a tumor. When the labeled antibody is internalized, the toxin is cleaved from the antibody in the cell’s endocytic compartment, allowing the toxin to penetrate and kill the cell.
The immunophilins are a family of intracellular proteins that bind to certain compounds forming complexes that interfere with signaling pathways important for the clonal expansion of lymphocytes.

67
Q

Certain microorganisms that infect the respiratory tract directly depress ciliary activity. This inhibits their removal from the lung and promotes infection. Bordetella pertussis attaches to respiratory epithelial cells and interferes with ciliary activity, causing the disease known as whooping cough. The organism produces a toxin that is an important virulence component and is an example of the group of toxins known as what?

1 Binary toxins
2 RTX toxins
3 Immunotoxins
4 Immunophilins
5 Endotoxins

A

RTX toxins

Toxins are significant virulence factors for bacteria. The table lists some bacteria that produce toxins and the diseases they cause (refer to the table).
Disease Organism Toxin
Listeriosis Listeria monocytogenes Listeriolysin
Pneumonia Streptococcus pneumoniae Pneumolysin
Tetanus Clostridium tetani Tetanus toxin
Diphtheria Corynebacterium diphtheriae Diphtheria toxin
Botulism Clostridium botulinum Botulinum toxin
Gas gangrene Clostridium perfringens Clostridial toxin
Protein toxins are secreted by gram-positive and gram-negative bacteria or released upon lysis from these microbes. One group of toxins has been designated RTX toxins(repeats in toxin) by virtue of a common structural feature: the presence of an array of a 9 amino acid repeat to which calcium binds, thereby activating the toxins which form membrane pores of varying sizes. They constitute the largest group of bacterial pore-forming toxins and are widespread among gram-negative pathogens.
Binary toxins are comprised of 2 proteins, only 1 of which is toxic; however, the other is necessary at some stage for manifestation of toxicity. Anthrax toxin is one example, and two others are Serratia and Proteuscytolysins and staphylococcal leukocidins.
Linking a tumor-specific or tumor-selective monoclonal antibody to a toxin is a way to create an immunotoxin, a reagent that can by used to destroy all or part of a tumor. When the labeled antibody is internalized, the toxin is cleaved from the antibody in the cell’s endocytic compartment, allowing the toxin to penetrate and kill the cell.
The immunophilins are a family of intracellular proteins that bind to certain compounds, forming complexes that interfere with signaling pathways important for the clonal expansion of lymphocytes.
Endotoxins are part of the outer membrane of gram-negative bacteria. Some of the diseases in which endotoxins may play a part are typhoid fever, tularemia, plague and brucellosis, and a variety of hospital-acquired infections caused by opportunistic gram-negative pathogens. Endotoxin is a complex lipopolysaccharide that displays a large array of biological effects.

68
Q

Botulism is caused by Clostridium botulinum, a widespread saprophyte present in soil and vegetable materials. This bacterium contaminates food, particularly inadequately preserved meat or vegetables. It produces a powerful neurotoxin. Clostridium botulinum C2 toxin is an example of a group of toxins known as what?

1 RTX toxins
2 Binary toxins
3 Immunotoxins
4 Immunophilins
5 Endotoxins

A

Binary toxins

Toxins are significant virulence factors for bacteria. The table lists some bacteria that produce toxins and the diseases they cause (refer to the table).
Disease Organism Toxin
Listeriosis Listeria monocytogenes Listeriolysin
Pneumonia Streptococcus pneumoniae Pneumolysin
Tetanus Clostridium tetani Tetanus toxin
Diphtheria Corynebacterium diphtheriae Diphtheria toxin
Botulism Clostridium botulinum Botulinum toxin
Gas gangrene Clostridium perfringens Clostridial toxin
Clostridium botulinum C2 toxin is an example of a group of toxins that also includes Clostridium perfringens iota toxin and Clostridium spiroforme toxin. The subunits of these binary toxins act together to produce their toxicity. Clostridium toxin is absorbed from the intestine and acts on the peripheral nervous system, interfering with the release of acetylcholine at cholinergic synapses of neuromuscular junctions. Somewhere between 12 and 36 hours after ingestion there are clinical signs suggesting an acute neurological disorder, with vertigo, cranial nerve palsies, and finally death a few days later with respiratory failure. About 145 cases of this intoxication, called botulism, are reported annually to the CDC.
One group of toxins has been designated RTX toxins (repeats in toxin) by virtue of a common structural feature: the presence of an array of a 9 amino acid repeat to which calcium binds, thereby activating the toxins, which form membrane pores of varying sizes. They constitute the largest group of bacterial pore-forming toxins and are widespread among gram-negative pathogens.
Linking a tumor-specific or tumor-selective monoclonal antibody to a toxin is a way to create an immunotoxin, a reagent that can be used to destroy all or part of a tumor. When the labeled antibody is internalized, the toxin is cleaved from the antibody in the cell’s endocytic compartment, allowing the toxin to penetrate and kill the cell.
The immunophilins are a family of intracellular proteins that bind to certain compounds, forming complexes that interfere with signaling pathways important for the clonal expansion of lymphocytes.
Endotoxins are part of the outer membrane of gram-negative bacteria. Some of the diseases in which endotoxins may play a part are typhoid fever, tularemia, plague and brucellosis, and a variety of hospital-acquired infections caused by opportunistic gram-negative pathogens. Endotoxin is a complex lipopolysaccharide that displays a large array of biological effects.

69
Q

Serratia marcescens is an opportunistic pathogen that causes a range of infections including respiratory and urinary tract infections. Proteus mirabilis causes acute pyelonephritis. Pathogenicity is multi-factorial but the accumulated evidence is that a membrane-active cytolysin plays an important role in disease causation by both pathogens. This toxin is part of a group known as what?

1 RTX toxins
2 Immunotoxins
3 Binary toxins
4 Immunophilins
5 Endotoxins

A

Binary toxins

Toxins are significant virulence factors for bacteria. The table lists some bacteria that produce toxins and the diseases they cause (refer to the table).
Disease Organism Toxin
Listeriosis Listeria monocytogenes Listeriolysin
Pneumonia Streptococcus pneumoniae Pneumolysin
Tetanus Clostridium tetani Tetanus toxin
Diphtheria Corynebacterium diphtheriae Diphtheria toxin
Botulism Clostridium botulinum Botulinum toxin
Gas gangrene Clostridium perfringens Clostridial toxin
Protein toxins are secreted by gram-positive and gram-negative bacteria or released upon lysis from these microbes. Some of them are enzymes. Binary toxins are comprised of two proteins, only one of which is toxic but the other is necessary at some stage for manifestation of toxicity. S. marcescens hemolysin (ShlA) and P. mirabilis hemolysins (HpmA) represent a new type of cytolytic toxin: they are described as “cell-associated” and have a specialized means of delivery from the bacterial cell to the target host cell. ShlA and HpmA each require a second protein, ShlB and HpmB respectively. The B proteins form pores in the bacterial outer membrane facilitating the secretion of the corresponding A components, their concomitant activation and insertion into eukaryotic membranes. The A proteins are also cytotoxic, causing vacuolation in cells and release of a range of inflammatory mediators.
One group of toxins has been designated RTX toxins (repeats in toxin) by virtue of a common structural feature - the presence of an array of a nine amino acid repeat to which calcium binds, thereby activating the toxins, which form membrane pores of varying sizes. They constitute the largest group of bacterial pore-forming toxins and are widespread among gram-negative pathogens.
Linking a tumor-specific or tumor-selective monoclonal antibody to a toxin is a way to create an immunotoxin, a reagent that can by used to destroy all or part of a tumor. When the labeled antibody is internalized, the toxin is cleaved from the antibody in the cell’s endocytic compartment, allowing the toxin to penetrate and kill the cell.
The immunophilins are a family of intracellular proteins that bind to certain compounds forming complexes that interfere with signaling pathways important for the clonal expansion of lymphocytes.
Endotoxins are part of the outer membrane of gram-negative bacteria. Some of the diseases in which endotoxins may play a part are typhoid fever, tularemia, plague and brucellosis, and a variety of hospital-acquired infections caused by opportunistic gram-negative pathogens. Endotoxin is a complex lipopolysaccharide that displays a large array of biological effects.

70
Q

Helicobacter pylori is a specific human pathogen affecting billions of people worldwide. It is transmitted through the oro-fecal route, and colonizes the (seemingly inhospitable) niche of the stomach. Some 20% of infected patients can develop ulcers, and infected individuals have an increased risk of stomach cancer. What is a toxin associated with this microbe?

1 Urease
2 Streptolysin
3 Immunotoxin
4 Immunophilin
5 Endotoxin

A

Urease

Protein toxins are secreted by, or released upon lysis from, both gram-positive and gram-negative bacteria, and are historically referred to as exotoxins. They are proteins, some of which are enzymes. When liberated locally, they can cause cell and tissue damage. An essential virulence factor of H. pylori is a potent urease that is synthesized in vast quantity by the organism, and released by autolysis and efficiently absorbed onto the surface of viable organisms. Urease catalyzes the hydrolysis of urea to form carbon dioxide and ammonia. It is important in local neutralization of stomach acidity, thereby allowing H. pylori to penetrate the protective mucus layer overlying the lining of the stomach, where the organism attaches to gastric epithelial cells.
Some bacteria, as they multiply in tissues, release soluble materials that are lethal for phagocytes. Pathogenic streptococci release hemolysins, or streptolysins, that lyse red blood cells and have an important toxic action on polymorphs and macrophages.
Linking a tumor-specific or tumor-selective monoclonal antibody to a toxin is a way to create an immunotoxin, a reagent that can be used to destroy all, or part, of a tumor. When the labeled antibody is internalized, the toxin is cleaved from the antibody in the cell’s endocytic compartment, allowing the toxin to penetrate and kill the cell.
The immunophilins are a family of intracellular proteins that bind to certain compounds, forming complexes that interfere with signaling pathways important for the clonal expansion of lymphocytes.
Endotoxins are part of the outer membrane of gram-negative bacteria. Some of the diseases in which endotoxins may play a part are typhoid fever, tularemia, plague, brucellosis, and a variety of hospital-acquired infections caused by opportunistic gram-negative pathogens. Endotoxin is a complex lipopolysaccharide that displays a large array of biological effects.

71
Q

Plague is one of the most deadly diseases of man, and has, over several thousand years, claimed millions of lives. The causative agent of the plague, Yersinia pestis, is primarily a parasite of rodents in which it is endemic in many areas of the world. Only when man comes in close proximity with infected rodents do outbreaks of human plague occur. The disease is spread from rat to rat and from rat to man by fleas. The symptoms of plague - high fever and vascular damage - are characteristics of intoxication with what toxin?

1 Binary toxin
2 RTX toxin
3 Endotoxin
4 Immunotoxin
5 Immunophilin

A

Endotoxin

Protein toxins are secreted by gram-positive and gram-negative bacteria or released upon lysis from these microbes. Endotoxins are part of the outer membrane of gram-negative bacteria. Some of the diseases in which endotoxins may play a part are typhoid fever, tularemia, plague and brucellosis, and a variety of hospital-acquired infections caused by opportunistic gram-negative pathogens. Endotoxin is a complex lipopolysaccharide (LPS) that displays a large array of biological effects. When large amounts of the molecule enter the blood, there are profound effects on blood vessels with peripheral vascular pooling, a drastic fall in blood pressure, collapse, and sometimes death.
Binary toxins are comprised of 2 proteins, only 1 of which is toxic; however, the other is necessary at some stage for manifestation of toxicity. Examples of binary toxins are Serratia and Proteus cytolysins and staphylococcal leukocidins.
One group of toxins has been designated RTX toxins (repeats in toxin) by virtue of a common structural feature: the presence of an array of a 9 amino acid repeat to which calcium binds, thereby activating the toxins, which form membrane pores of varying sizes. They constitute the largest group of bacterial pore-forming toxins and are widespread among gram-negative pathogens.
Linking a tumor-specific or tumor-selective monoclonal antibody to a toxin is a way to create an immunotoxin, a reagent that can by used to destroy all or part of a tumor. When the labeled antibody is internalized, the toxin is cleaved from the antibody in the cell’s endocytic compartment, allowing the toxin to penetrate and kill the cell.
The immunophilins are a family of intracellular proteins that bind to certain compounds, forming complexes that interfere with signaling pathways important for the clonal expansion of lymphocytes.

72
Q

Many important pathogenic bacteria bear on their surface substances that inhibit phagocytosis. Resistance to phagocytosis is sometimes due to a component of the bacterial cell wall, and sometimes it is due to a capsule enclosing the bacterial wall, secreted by the bacterium. What is a classical example of an antiphagocytic substance on the bacterial surface?

1 PHA
2 M protein
3 C-reactive protein
4 Proteasome
5 Agretope

A

M protein

Microbial products that kill phagocytes may at lower concentrations interfere with their locomotion or their phagocytic activity. A more direct challenge to the phagocyte is provided by the various microorganisms whose surface properties prevent their phagocytosis. The M proteins, fibrous structures of streptococci and the polysaccharide capsules of pneumococci are examples of substances blocking phagocytosis. Streptococci and pneumococci owe their success to their ability to survive and grow extracellularly, avoiding uptake by phagocytic cells. Certain M proteins on the surface and the pili of streptococci are undoubtedly associated with resistance to phagocytosis and with virulence.
Mitogen-stimulated lymphocyte activation triggers biochemical signaling pathways that lead to gene expression, protein synthesis, cell proliferation, and differentiation. Several sugar-binding proteins called lectins are very effective mitogens. Phytohemagglutinin or PHA is a lectin that simulates T lymphocytes but not B-lymphocytes.
C-reactive protein (CRP) binds to the phosphorylcholine portion of bacterial and fungal cell wall lipopolysaccharides. Phosphorylcholine is also found in mammalian cells membrane phospholipids but in a form that cannot react with C-reactive protein. When C-reactive protein binds to a bacterium, it is not only able to opsonize it but can also activate the complement cascade by binding to C1q, the first component of the classical pathway of complement activation.
The proteasome is a large cylindrical complex of some 28 subunits, arranged as four stacked rings, each of seven subunits, and it has a hollow core lined by the active sites of the proteolytic subunits of the proteasome. The proteasome cuts protein into peptide fragments eight to nine residues long, and these peptides are selectively transported into the endoplasmic reticulum by the products of transporter genes.
The size of an epitope that binds to a specific T cell is about eight to 15 amino acid residues and it contains an area that binds to the MHC proteins of the antigen-presenting cell, an area called the agretope.

73
Q

Various substances released by bacteria influence the spread of infection. There are two bacterial toxins that can affect the physical arrival of PMNs to an infection site. The A and B toxins of C. difficile disrupt tight junctions in gut epithelia, thereby increasing the ease with which inflammatory cells arrive at the site. Another toxin, one that is released by E. coli actually helps maintain these tight junctions. What is it known as?

1 CAM-1
2 M protein
3 CNF 1
4 Mac-1
5 LFA-1

A

CNF 1

CNF 1 or Cytotoxic Necrotizing Factor 1 is a heat-labile, monomeric 115 kDa protein encoded by a gene on the chromosome of pathogenic E. coli. It acts on Rho GTPases, which participate in the regulation of the actin cytoskeleton in normal cells. CNF 1 deaminates glutamine 63 in RhoA, changing it to glutamate. This increases stress fibers in the target cell, causing it to enlarge and become syncytial. The cell becomes multinucleated and dies. The toxin is thought to maintain the closure of tight junctions in the epithelium of the gut and thus decreases the numbers of PMNs arriving at the focus of infection by blocking their traverse through the paracellular route. CNF 1 is often associated with intestinal and extraintestinal infections and plays a role in diarrhea and urinary tract infections.
ICAM-1 is one of the principal ligands for the leukocyte β2-integrins. It was the first ICAM to be described, and is present on leukocytes and endothelial cells, but it is also expressed in several other tissues. It is upregulated during inflammation - it is induced by cytokines such as tumor necrosis factor-α, interferons, etc.
Microbial products that kill phagocytes may at lower concentrations interfere with their locomotion or their phagocytic activity. A more direct challenge to the phagocyte is provided by the various microorganisms whose surface properties prevent their phagocytosis. The M proteins, fibrous structures of streptococci and the polysaccharide capsules of pneumococci are examples of substances blocking phagocytosis.
Mac-1, also called CD11b/CD18, is stored in specific granules that are shuttled to the granulocyte surface. It exists as a chemoattractant activation-dependent molecule that undergoes a conformational change upon stimulation. Until stimulation occurs, it remains in a resting, non-adhesive state. Mac-1 is a β2 integrin.
LFA-1 is a β2 integrin that is critical in neutrophil transmigration and important in transmigration of other leukocyte subtypes. Stimulation by locally bound chemokines activates LFA-1 on the T cell, increasing its affinity for ICAM-2, which is expressed constitutively on all endothelial cells. The binding of LFA-1 to its ligands ICAM-1 and ICAM-2 has a major role in T cell adhesion to and migration through the wall of the blood vessel into the lymph node.

74
Q

Direct spread of microorganisms through normal subepithelial tissues is not easy. This spread is limited physically by the gel-like nature of the connective tissue matrix. Certain bacteria produce soluble substances with an effect on the physical properties of this connective tissue matrix, and these substances can promote the spread of infection. For example, invasive streptococci produce

1 Urease
2 Streptolysin
3 Immunotoxin
4 Immunophilin
5 Hyaluronidase

A

Hyaluronidase

Bacteria produce a great variety of enzymes, including proteinases, collagenases, lipases, and nucleases, but only very few of them have been clearly shown to be of any pathogenic significance. Invasive streptococcal species liberate hyaluronidase, an enzyme that liquefies the hyaluronic acid component of the connective tissue matrix. Streptococcal skin infections, accordingly, often spread rapidly through the dermis causing the condition erysipelas. Hyaluronidase was originally referred to as the “spreading factor,” but it is not a necessary factor, because bacteria such as Brucella are highly invasive in the absence of hyaluronidase.
An essential virulence factor of H. pylori is a potent urease that is synthesized in vast quantity by the organism, and released by autolysis and efficiently absorbed onto the surface of viable organisms. It is important in local neutralization of stomach acidity, thereby allowing H. pylori to penetrate the protective mucus layer overlying the lining of the stomach where the organism attaches to gastric epithelial cells.
Some bacteria, as they multiply in tissues, release soluble materials that are lethal for phagocytes. Pathogenic streptococci release hemolysins or streptolysins that lyse red blood cells and have an important toxic action on polymorphs and macrophages.
Linking a tumor-specific or tumor-selective monoclonal antibody to a toxin is a way to create an immunotoxin, a reagent that can by used to destroy all or part of a tumor. When the labeled antibody is internalized, the toxin is cleaved from the antibody in the cell’s endocytic compartment, allowing the toxin to penetrate and kill the cell.
The immunophilins are a family of intracellular proteins that bind to certain compounds forming complexes that interfere with signaling pathways important for the clonal expansion of lymphocytes.

75
Q

Streptococcus pneumoniae, or pneumococcus, is often found as part of the normal upper respiratory tract flora. As an opportunist it can spread to infected or damaged lungs and cause pneumonia. It can also cause otitis media and meningitis. The polysaccharide capsule of the pneumococcus is associated with resistance to

1 Neutralization
2 Pinocytosis
3 Endocytosis
4 Phagocytosis
5 Apoptosis

A

Phagocytosis

It is not clear why the polysaccharide capsule of S. pneumoniae confers resistance to phagocytosis. Possible its slimy polysaccharide nature makes the phagocytic act difficult for purely mechanical reasons. Although antibody is needed for phagocytosis in a fluid medium, it is known that phagocytosis takes place without antibody on the solid surface lining of an alveolus or lymphatic vessel where the physical act of phagocytosis is favored, and the phagocyte can corner and get around the bacterium. Pathogenic bacteria with similar polysaccharide capsules include Haemophilus influenzae and Klebsiella pneumoniae.
A simple and direct way in which antibodies can protect from pathogens or their toxic products is by binding to them and thereby blocking their access to cells that they may infect or destroy. This is known as neutralization and is important for protection against bacterial toxins and against pathogens such as viruses, which can thus be prevented from entering cells and replicating.
Pinocytosis is the simple cellular uptake of fluid and solutes. It is a much simpler process than phagocytosis.
Some enveloped viruses enter the cell using endocytosis, the invagination of the plasma membrane into smaller virus-sized depressions coated on the cytoplasmic side with a cellular protein known as clathrin. These detach from the plasma membrane and become vesicles free in the cell’s cytoplasm.
Apoptosis represents physiological as opposed to pathological (necrotic) cell death - it is the cellular equivalent of suicide. Apoptotic cells are recognized by phagocytes and removed.

76
Q

When Listeria monocytogenes infects macrophages, a portion of incoming bacteria escapes into the cytoplasm. The bacteria are taken into phagosomes that are acidified, a condition that is necessary for the activity of a vital virulence determinant mediating escape from the vacuole. What is this virulence determinant known as?

1 Binary toxin
2 Listeriolysin
3 Toxoid
4 Streptolysin
5 Pilin

A

Listeriolysin

Listeriolysin is the most important virulence determinant of Listeria monocytogenes. It plays an important part in mediating the escape of the bacteria from intraphagocytic vacuoles. Exogenous addition of listeriolysin will rapidly kill cells by rupturing the cytoplasmic membranes. It is also a potent trigger of host cell-signaling molecules. Many of the responses elicited by listeriolysin are believed to be the result of the activation of cytosolic NF-κB (host-cell stress-inducible transcription factor) and its translocation into the nucleus where it acts as a transcriptional activator of different genes involved in the immune response.
Binary toxins are comprised of two proteins, only one of which is toxic but the other is necessary at some stage for manifestation of toxicity. Anthrax toxin is one example, and two more are Serratia and Proteus cytolysins and staphylococcal leukocidins.
Diphtheria and tetanus toxins are among the bacterial toxins in which the toxic and the receptor-binding functions of the molecule are on two separate chains. Toxoids are modified toxin molecules that lack toxic activity but retain the receptor-binding site, so that immunization with the toxoid induces neutralizing antibodies effective in protecting against the native toxin.
Some bacteria, as they multiply in tissues, release soluble materials that are lethal for phagocytes. Pathogenic streptococci release hemolysins or streptolysins that lyse red blood cells and have an important toxic action on polymorphs and macrophages.
Pili are threads or fine hairs protruding from bacterial surfaces. They assist bacteria in establishing colonies and can be conferred on bacteria by plasmids. Pilin is the protein subunit of pili and consists of constant, variable, and hypervariable regions analogous with immunoglobulin molecules.

77
Q

Case
A 65-year-old man presents with watery diarrhea and mild abdominal pain. He has had 8-9 non-bloody bowel movements per day for the last 10 days; he currently feels a general abdominal discomfort. On examination, his pulse rate, blood pressure, and oral temperature are normal. There is no abdominal tenderness. The stool guaiac test is negative. He does not travel. He eats home-cooked meals most of the time, and he reports using water from his private well for all of his needs.

In addition to consulting the EPA for decontamination advice, what recommendation would you provide this patient?

1 Close the well and never use it again.
2 Boil water before using.
3 Filter water before using.
4 Chlorinate water before using.
5 Send samples of water for lab analysis.

A

Boil water before using.

The water should be boiled before use and the patient should ask the US Environmental Protection Agency for advice on well decontamination and safe maintenance.
The symptoms are most likely caused by infection with an enterotoxigenic strain of E. coli, the bacteria commonly associated with gastroenteritis. This is also known as traveler’s diarrhea. The patient did not travel, but he drinks water from a private well. His well is most likely contaminated. Coliforms are naturally present in the environment, and pathogenic strains can occasionally infect water sources if they are not well maintained. Boiling the water is sufficient to free it from such contaminants.
Drastic measures like closing the well are only required if contamination is recurring and the water does not become clean and safe to drink after following standard decontamination procedures.
Most in-home available filters do not remove all bacteria and viruses from water. This method is less effective than boiling. Chlorine, UV-radiation, and ozone are the three main water decontaminants, but strict procedures have to be followed. It is not advised to drink highly chlorinated water.
Analysis of the well water should be performed as part of a regular maintenance procedure to ensure the water is safe to drink, not as a solution to these specific circumstances.

78
Q

Case
A 55-year-old woman who has been in the intensive care unit for 3 weeks following a pelvic fracture develops a urinary tract infection with a multiple drug resistant (MDR), catalase-negative, gram-positive coccus. Within the next 2 weeks, 3 other intensive care unit patients develop infections with the same organism. As part of infection control measures, surveillance cultures are carried out to detect patients colonized by the suspected nosocomial pathogen.

What specimen is ideal for this purpose?

1 Nasal swab
2 Skin swab
3 Blood sample
4 Urine sample
5 Rectal swab

A

Rectal swab

The major MDR gram-positive cocci associated with outbreaks of nosocomial infections are methicillin-resistant Staphylococcus Aureus (MRSA) and vancomycin-resistant Enterococcus (VRE). Of these, catalase-negative, gram-positive coccus indicates VRE. VRE colonization primarily occurs in the lower gastrointestinal tract and frequently precedes infection; therefore, of the listed options, a rectal swab is the ideal specimen to be collected for detecting patients colonized by this pathogen.
MRSA colonizes in various body sites. Frequent colonization occurs in the anterior nares, and this makes nasal swabbing a good choice for MRSA surveillance. The skin may also be colonized by MRSA, so skin swabbing can be done.
Urine and blood samples are tested when there is clinical suspicion of infection; however, because enterococcus does not colonize in the bladder or blood, these are not good sites to use to detect colonization.